gmat essay question examples

Homepage > Prepare for GMAT > GMAT Essay – List of AWA Topics – 50 Practice Questions on GMAT AWA 2023

GMAT Essay – List of AWA Topics – 50 Practice Questions on GMAT AWA 2023

Posted by Suheb Hussain | Jan 25, 2023 | GMAT IR and AWA , Know the GMAT , Prepare for GMAT

GMAT Essay – List of AWA Topics – 50 Practice Questions on GMAT AWA 2023

This document contains practice questions that will help you improve the AWA section or the GMAT essay section. Discuss in your GMAT essay how well-reasoned you find the argument. Here are some ways you can do that while writing your GMAT essay for the AWA section. In your discussion, be sure to analyze the line of reasoning and the use of evidence in the argument. For example, you may need to consider what questionable assumptions underlie the thinking and what alternative explanations or counterexamples might weaken the conclusion. You can also discuss what sort of evidence would strengthen or refute the argument, what changes in the argument would make it more logically sound, and what, if anything, would help you better evaluate its conclusion.

We also recommend you read this article on how to score a perfect 6 on the GMAT AWA.

GMAT Essay AWA Practice Questions

Begin your GMAT preparation with the only prep company that has delivered more 700+ scores than any other GMAT club partner . Achieve GMAT 740+ with our AI driven tools that you personalized feedback at every step of your GMAT journey. Take our free trial today!

GMAT Essay practice question 1

  • The following appeared as part of an article in a trade magazine:

“During a recent trial period in which government inspections at selected meat-processing plants were more frequent, the number of bacteria in samples of processed chicken decreased by 50 percent on average from the previous year’s level. If the government were to institute more frequent inspections, the incidence of stomach and intestinal infections throughout the country could thus be cut in half. In the meantime, consumers of Excel Meats should be safe from infection because Excel’s main processing plant has shown more improvement in eliminating bacterial contamination than any other plant cited in the government report.”

Discuss how well reasoned . . . etc.

GMAT Essay practice question 2

2. The following appeared as part of an annual report sent to stockholders by Olympic Foods, a processor of frozen foods:

“Over time, the costs of processing go down because as organizations learn how to do things better, they become more efficient. In color film processing, for example, the cost of a 3-by-5-inch print fell from 50 cents for five-day service in 1970 to 20 cents for one-day service in 1984. The same principle applies to the processing of food. And since Olympic Foods will soon celebrate its 25th birthday, we can expect that our long experience will enable us to minimize costs and thus maximize profits.”

GMAT AWA practice question 3

3. The following appeared in a memorandum from the business department of the Apogee Company:

“When the Apogee Company had all its operations in one location, it was more profitable than it is today. Therefore, the Apogee Company should close down its field offices and conduct all its operations from a single location. Such centralization would improve profitability by cutting costs and helping the company maintain better supervision of all employees.”

If you are planning to take the GMAT, we can help you with a personalized study plan and give you access to quality online content to prepare. Write to us at  [email protected] . We are the  most reviewed GMAT prep company on gmatclub with more than 2500 reviews. Why don’t you take a free trial and judge for yourself?

GMAT Essay practice question 4

4. The following appeared in a memorandum issued by a large city’s council on the arts:

“In a recent citywide poll, 15 percent more residents said that they watch television programs about the visual arts than was the case in a poll conducted five years ago. During these past five years, the number of people visiting our city’s art museums has increased by a similar percentage. Since the corporate funding that supports public television, where most of the visual arts programs appear, is now being threatened with severe cuts, we can expect that attendance at our city’s art museums will also start to decrease. Thus some of the city’s funds for supporting the arts should be reallocated to public television.”

GMAT Essay practice question 5

5. The following appeared in an announcement issued by the publisher of The Mercury , a weekly newspaper:

“Since a competing lower-priced newspaper, The Bugle , was started five years ago, The Mercury ’s circulation has declined by 10,000 readers. The best way to get more people to read The Mercury is to reduce its price below that of The Bugle , at least until circulation increases to former levels. The increased circulation of The Mercury will attract more businesses to buy advertising space in the paper.”

Attention test takers! GMAC has recently announced a new version of the GMAT test called the GMAT Focus edition. This new test will be launched in the latter part of 2023, while the current version of the test will be available till at least early 2024.

Read this article to know the key differences between the current version and the GMAT focus edition .

Also, watch the following video to learn all about the GMAT Focus edition and also understand how to decide which version of the exam to take- The current version or the GMAT Focus edition.

GMAT Essay practice question 6

6.  The following appeared in a report presented for discussion at a meeting of the directors of a company that manufactures parts for heavy machinery:

“The falling revenues that the company is experiencing coincide with delays in manufacturing. These delays, in turn, are due in large part to poor planning in purchasing metals. Consider further that the manager of the department that handles purchasing of raw materials has an excellent background in general business, psychology, and sociology, but knows little about the properties of metals. The company should, therefore, move the purchasing manager to the sales department and bring in a scientist from the research division to be manager of the purchasing department.”

GMAT Essay practice question 7

7. The following appeared in a research paper written for an introductory economics course:

“For the past century, an increase in the number of residential building permits issued per month in a particular region has been a reliable indicator of coming improvements to that region’s economy. If the monthly number of residential building permits issued rises consistently for a few months, the local unemployment rate almost always falls and economic production increases. This well-established connection reveals an effective method by which a regional government can end a local economic downturn: relax regulations governing all construction so that many more building permits can be issued.”

Discuss how well reasoned . . .etc.

GMAT Essay practice question 8

8.  The following appeared in a memorandum to the work-group supervisors of the GBS Company:

“The CoffeeCart beverage and food service located in the lobby of our main office building is not earning enough in sales to cover its costs, and so the cart may discontinue operating at GBS. Given the low staff morale, as evidenced by the increase in the number of employees leaving the company, the loss of this service could present a problem, especially since the staff morale questionnaire showed widespread dissatisfaction with the snack machines.

Therefore, supervisors should remind the employees in their group to patronize the cart—after all, it was leased for their convenience so that they would not have to walk over to the cafeteria on breaks.”

GMAT Essay practice question 9

9. The following appeared in a memorandum written by the chair of the music department to the president of Omega University:

“Mental health experts have observed that symptoms of mental illness are less pronounced in many patients after group music-therapy sessions, and job openings in the music-therapy field have increased during the past year. Consequently, graduates from our degree program for music therapists should have no trouble finding good positions. To help improve the financial status of Omega University, we should, therefore, expand our music-therapy degree program by increasing its enrollment targets.”

GMAT Essay practice question 10

10. The following appeared in a memorandum from the vice president of Gigantis, a development company that builds and leases retail store facilities:

“Nationwide over the past five years, sales have increased significantly at outlet stores that deal exclusively in reduced-price merchandise. Therefore, we should publicize the new mall that we are building at Pleasantville as a central location for outlet shopping and rent storage space only to outlet companies. By taking advantage of the success of outlet stores, this plan should help ensure full occupancy of the mall and enable us to recover quickly the costs of building the mall.”

11. The following appeared in a memorandum from the business planning department of Avia Airlines:

“Of all the cities in their region, Beaumont and Fletcher are showing the fastest growth in the number of new businesses. Therefore, Avia should establish a commuter route between them as a means of countering recent losses on its main passenger routes. And to make the commuter route more profitable from the outset, Avia should offer a 1/3 discount on tickets purchased within two days of the flight. Unlike tickets bought earlier, discount tickets will be nonrefundable, and so gain from their sale will be greater.”

12. The following appeared in a speech by a stockholder of Consolidated Industries at the company’s annual stockholders’ meeting:

“In the computer hardware division last year, profits fell significantly below projections, the product line decreased from 20 to only 5 items, and expenditures for employee benefits increased by 15 percent. Nevertheless, Consolidated’s board of directors has approved an annual salary of more than $1 million for our company’s chief executive officer. The present board members should be replaced because they are unconcerned about the increasing costs of employee benefits and salaries, in spite of the company’s problems generating income.”

13.  The following appeared as part of a business plan by the Capital Idea investment firm:

“In recent years the worldwide demand for fish has grown, and improvements in fishing technology have made larger catches and thus increased supply possible: for example, last year’s tuna catch was 9 percent greater than the previous year’s. To capitalize on these trends, we should, therefore, invest in the new tartfish processing plant on Tartfish Island, where increasing revenues from tourism indicate a strong local economy.”

14. The following appeared as part of an article in a weekly newsmagazine:

“The country of Oleum can best solve the problem of its balance of trade deficit by further increasing the tax on its major import, crude oil. After Oleum increased the tax on imported crude oil four months ago, consumption of gasoline declined by 20 percent. Therefore, by imposing a second and significantly higher tax increase next year, Oleum will dramatically decrease its balance of trade deficit.”

15. The following appeared in a memorandum from the human resources department of HomeStyle, a house remodeling business:

“This year, despite HomeStyle’s move to new office space, we have seen a decline in both company morale and productivity, and a corresponding increase in administrative costs. To rectify these problems, we should begin using a newly developed software package for performance appraisal and feedback. Managers will save time by simply choosing comments from a preexisting list; then the software will automatically generate feedback for the employee. The human resources department at CounterBalance, the manufacturer of the countertops we install, reports satisfaction with the package.”

16. The following appeared in a memorandum written by the managing director of the Exeunt Theater Company:

“Now that we have moved to a larger theater, we can expect to increase our revenues from ticket sales. To further increase profits, we should start producing the plays that have been most successful when they were performed in our nation’s largest cities. In addition, we should hire the Adlib Theater Company’s director of fund-raising, since corporate contributions to Adlib have increased significantly over the three years that she has worked for Adlib.”

Do you also want a 700+ on the GMAT? Try out our Free Trial today and get access to 10+ hours of free interactive video lessons, 400+ GMAT-like questions, and AI-drive tools that help you track your progress. We are the most reviewed GMAT Preparation company on the GMAT Club , with 2500+ reviews!

17. The following appeared in a memorandum from a regional supervisor of post office operations:

“During a two-week study of postal operations, the Presto City post office handled about twice as many items as the Lento City post office, even though the cities are about the same size. Moreover, customer satisfaction appears to be higher in Presto City, since the study found fewer complaints regarding the Presto City post office. Therefore, the postmasters at these two offices should exchange assignments: the Presto City postmaster will solve the problems of inefficiency and customer dissatisfaction at the Lento City office while the Lento City postmaster learns firsthand the superior methods of Presto City.”

18. The following appeared in a memorandum from the human resources department of Diversified Manufacturing:

“Managers at our central office report that their employees tend to be most productive in the days immediately preceding a vacation. To help counteract our declining market share, we could increase the productivity of our professional staff members, who currently receive four weeks paid vacation a year, by limiting them to a maximum of one week’s continuous vacation time. They will thus take more vacation breaks during a year and give us more days of maximum productivity.”

19. The following appeared in a memorandum from the marketing department of a children’s clothing manufacturer:

“Our HuggyBunny brand is the bestselling brand of children’s clothing. Parents everywhere recognize the HuggyBunny logo as a mark of quality, and most of our customers show great brand loyalty. Sales reports have shown that parents are more likely to buy children’s clothes with the familiar HuggyBunny brand and logo than otherwise identical clothes without it. Therefore, if we use the HuggyBunny brand name and logo for the new line of clothing for teenagers that our company will soon be introducing, that clothing will sell better than it would if we labeled it with a new brand name and logo.”

20. The following appeared in a memorandum from the president of Aurora, a company that sells organic milk (milk produced without the use of chemical additives):

“Sales of organic food products in this country have tripled over the past five years. If Aurora is to profit from this continuing trend, we must diversify and start selling products such as organic orange juice and organic eggs in addition to our regular product line. With the recent increase of articles in health magazines questioning the safety of milk and other food products, customers are even more likely to buy our line of organic products. And to help ensure our successful expansion, we should hire the founder of a chain of health-food stores to serve as our vice president of marketing.”

21. The following appeared as part of an article in a newsletter for farmers:

“Users of Solacium, a medicinal herb now grown mainly in Asia, report that it relieves tension and promotes deep sleep. A recent study indicates that a large number of college students who took pills containing one of the ingredients in Solacium suffered less anxiety. To satisfy the anticipated demands for this very promising therapeutic herb and to reap the financial benefits, farmers in this country should begin growing it.”

22. The following appeared as part of the business plan of the Capital Idea investment firm:

“Across town in the Park Hill district, the Thespian Theater, Pizzazz Pizza, and the Niblick Golf Club have all had business increases over the past two years. Capital Idea should, therefore, invest in the Roxy Playhouse, the Slice-o’- Pizza, and the Divot Golf Club, three new businesses in the Irongate district. As a condition, we should require them to participate in a special program: Any customer who patronizes two of the businesses will receive a substantial discount at the third. By motivating customers to patronize all three, we will thus contribute to the profitability of each and maximize our return.”

23. The following appeared in a memorandum from the owner of Carlo’s Clothing to the staff:

“Since Disc Depot, the music store on the next block began a new radio advertising campaign last year, its business has grown dramatically, as evidenced by the large increase in foot traffic into the store. While the Disc Depot’s owners have apparently become wealthy enough to retire, profits at Carlo’s Clothing have remained stagnant for the past three years. In order to boost our sales and profits, we should, therefore, switch from newspaper advertising to frequent radio advertisements like those for Disc Depot.”

24. The following appeared in a letter to the editor of a River City newspaper:

“The Clio Development Group’s plan for a multilevel parking garage on Dock Street should be approved in order to strengthen the economy of the surrounding area. Although most of the buildings on the block would have to be demolished, they are among the oldest in the city and thus of little current economic value. Those who oppose the project should realize that historic preservation cannot be the only consideration: even Athens or Jerusalem will knock down old buildings to put up new ones that improve the local economy.”

25. The following appeared in a memorandum from the publisher to the staff of The Clarion , a large metropolitan newspaper:

“During the recent campaign for mayor, a clear majority of city readers who responded to our survey indicated a desire for more news about city government. To increase circulation, and thus our profits, we should therefore consistently devote a greater proportion of space in all editions of The Clarion to coverage of local news.”

26. The following appeared in a corporate planning memorandum for a company that develops amusement parks:

“Because travel from our country to foreign countries has increased dramatically in recent years, our next project should be a ‘World Tour’ theme park with replicas of famous foreign buildings, rides that have international themes, and refreshment stands to serve only foods from the country represented by the nearest ride. The best location would be near our capital city, which has large percentages of international residents and of children under the age of 16. Given the advantages of this site and the growing interest in foreign countries, the ‘World Tour’ theme park should be as successful as our space-travel theme park, where attendance has increased tenfold over the past decade.”

27.  The following appeared in a letter to the editor of a River City newspaper:

“The Clio Development Group should not be permitted to build a multilevel parking garage on Dock Street since most of the buildings on the block would have to be demolished. Because these buildings were erected decades ago, they have historic significance and must, therefore, be preserved as economic assets in the effort to revitalize a restored riverfront area. Recall how Lakesburg has benefited from business increases in its historic downtown center. Moreover, there is plenty of vacant land for a parking lot elsewhere in River City.”

28. The following appeared in a letter to the editor of a local newspaper:

“Our city council’s neglect of the impoverished Railroad Flats neighborhood has left businesses with little incentive to locate there. Building a new professional football stadium in the neighborhood would solve this problem. Thousands of football fans would travel to the area to see games, and they would buy from local merchants, encouraging new businesses to open. So our city council should move quickly to fund the construction of a professional football stadium in Railroad Flats in order to help the neighborhood develop a thriving economy.”

29. The following appeared in a memorandum from the director of marketing for a pharmaceutical company:

“According to a survey of 5,000 urban residents, the prevalence of stress headaches increases with educational level, so that stress headaches occur most often among people with graduate-school degrees. It is well established that, nationally, higher educational levels usually correspond with higher levels of income. Therefore, in marketing our new pain remedy, Omnilixir, we should send free samples primarily to graduate students and to people with graduate degrees, and we should concentrate on advertising in professional journals rather than in general interest magazines.”

30. The following appeared as part of an editorial in the Waymarsh city newspaper:

“Last year the parents of first graders in our school district expressed satisfaction with the reading skills their children developed but complained strongly about their children’s math skills. To remedy this serious problem and improve our district’s elementary education, everyone in the teacher-training program at Waymarsh University should be required to take more courses in mathematics.”

31. The following appeared in a memorandum from the business office of the Lovin’ Cupful, a national restaurant chain:

“The Lovin’ Cupful franchises in our northeast region have begun serving customers Almost, a brand new powdered instant tea, in place of brewed tea. Waiters report that only about 2 percent of the customers have complained and that customers who want refills typically ask for ‘more tea.’ It appears, then, that 98 percent of the customers are perfectly happy with the switch, or else they cannot tell powdered instant from brewed tea. Therefore, in order to take advantage of the lower price per pound of Almost, all of our restaurants should begin substituting it for brewed tea.”

32. The following appeared as a memorandum from the vice-president of the Dolci candy company:

“Given the success of our premium and most expensive line of chocolate candies in a recent taste test and the consequent increase in sales, we should shift our business focus to producing additional lines of premium candy rather than our lower-priced, ordinary candies. When the current economic boom ends and consumers can no longer buy major luxury items, such as cars, they will still want to indulge in small luxuries, such as expensive candies.”

33. The following appeared in a memorandum from the director of research and development at Ready-to-Ware, a software engineering firm:

“The package of benefits and incentives that Ready-to-Ware offers to professional staff is too costly. Our quarterly profits have declined since the package was introduced two years ago, at the time of our incorporation. Moreover, the package had little positive effect, as we have had only marginal success in recruiting and training high-quality professional staff. To become more profitable again, Ready-to-Ware should, therefore, offer the reduced benefits package that was in place two years ago and use the savings to fund our current research and development initiatives.”

34. The following appeared in a memorandum from the assistant manager of Pageturner Books:

“Over the past two years, Pageturner’s profits have decreased by 5 percent, even though we have added a popular café as well as a music section selling CDs and tapes. At the same time, we have experienced an increase in the theft of merchandise. We should, therefore, follow the example of Thoreau Books, which increased its profits after putting copies of its most frequently stolen books on a high shelf behind the payment counter. By doing likewise with copies of the titles that our staff reported stolen last year, we too can increase profitability.”

35. The following appeared in a memorandum to a team developing accounting software for SmartPro Software, Inc.:

“Currently, more professional accountants use SmartPro accounting software than any other brand. However, in the market for personal accounting software for non-professionals to use in preparing their income tax returns, many of our competitors are outselling us. In surveys, our professional customers repeatedly say that they have chosen SmartPro Software because our most sophisticated software products include more advanced special features than competing brands. Therefore, the most effective way for us to increase sales of our personal accounting software for home users would clearly be to add the advanced special features that our professional software products currently offer.”

36. The following appeared in a memorandum written by the assistant manager of a store that sells gourmet food items from various countries:

“A local wine store made an interesting discovery last month: it sold more French than Italian wine on days when it played recordings of French accordion music, but it sold more Italian than French wine on days when Italian songs were played. Therefore, I recommend that we put food specialties from one particular country on sale for a week at a time and play only music from that country while the sale is going on. By this means we will increase our profits in the same way that the wine store did, and we will be able to predict more precisely what items we should stock at any given time.”

37. The following appeared in a memo to the Saluda town council from the town’s business manager:

“Research indicates that those who exercise regularly are hospitalized less than half as often as those who don’t exercise. By providing a well-equipped gym for Saluda’s municipal employees, we should be able to reduce the cost of our group health insurance coverage by approximately 50 percent and thereby achieve a balanced town budget.”

38. The following appeared as part of a letter to the editor of a local newspaper:

“Bayview High School is considering whether to require all of its students to wear uniforms while at school. Students attending Acorn Valley Academy, a private school in town, earn higher grades on average and are more likely to go on to college. Moreover, Acorn Valley reports few instances of tardiness, absenteeism, or discipline problems. Since Acorn Valley requires its students to wear uniforms, Bayview High School would do well to follow suit and require its students to wear uniforms as well.”

39. The following appeared as part of a memorandum from the loan department of the Frostbite National Bank:

“We should not approve the business loan application of the local group that wants to open a franchise outlet for the Kool Kone chain of ice cream parlors. Frostbite is known for its cold winters, and cold weather can mean slow ice cream sales. For example, even though Frostbite is a town of 10,000 people, it has only one ice cream spot—the Frigid Cow. Despite the lack of competition, the Frigid Cow’s net revenues fell by 10 percent last winter.”

40. The following appeared in a letter from a staff member in the office of admissions at Argent University:

“The most recent nationwide surveys show that undergraduates choose their major field primarily based on their perception of job prospects in that field. At our university, economics is now the most popular major, so students must perceive this field as having the best job prospects. Therefore, we can increase our enrollment if we focus our advertising and recruiting on publicizing the accomplishments of our best-known economics professors and the success of our economics graduates in finding employment.”

41. The following appeared as part of a business plan created by the management of the Take Heart Fitness Center:

“After opening the new swimming pool early last summer, Take Heart saw a 12 percent increase in the use of the center by its members. Therefore, in order to increase membership in Take Heart, we should continue to add new recreational facilities in subsequent years: for example, a multipurpose game room, a tennis court, and a miniature golf course. Being the only center in the area offering this range of activities would give us a competitive advantage in the health and recreation market.”

42. The following appeared as part of an article in the book section of a newspaper:

“Currently more and more books are becoming available in electronic form—either free-of-charge on the Internet or for a very low price-per-book on compact disc.* Thus literary classics are likely to be read more widely than ever before. People who couldn’t have purchased these works at bookstore prices will now be able to read them for little or no money; similarly, people who find it inconvenient to visit libraries and wait for books to be returned by other patrons will now have access to whatever classic they choose from their home or work computers. This increase in access to literary classics will radically affect the public taste in reading, creating a far more sophisticated and learned reading audience than has ever existed before.”

*A compact disc is a small portable disc capable of storing relatively large amounts of data that can be read by a computer.

43. The following appeared as an editorial in a magazine concerned with educational issues:

“In our country, the real earnings of men who have only a high-school degree have decreased significantly over the past 15 years, but those of male college graduates have remained about the same. Therefore, the key to improving the earnings of the next generation of workers is to send all students to college. Our country’s most important educational goal, then, should be to establish enough colleges and universities to accommodate all high school graduates.”

44. The following appeared in an editorial from a newspaper serving the town of Saluda:

“The Saluda Consolidated High School offers more than 200 different courses from which its students can choose. A much smaller private school down the street offers a basic curriculum of only 80 different courses, but it consistently sends a higher proportion of its graduating seniors on to college than Consolidated does. By eliminating at least half of the courses offered there and focusing on a basic curriculum, we could improve student performance at Consolidated and also save many tax dollars.”

45. The following appeared as part of an article in a trade magazine for breweries:

“Magic Hat Brewery recently released the results of a survey of visitors to its tasting room last year. Magic Hat reports that the majority of visitors asked to taste its low-calorie beers. To boost sales, other small breweries should brew low-calorie beers as well.”

46. The following appeared in a memorandum sent by a vice-president of the Nadir Company to the company’s human resources department:

“Nadir does not need to adopt the costly ‘family-friendly’ programs that have been proposed, such as part-time work, work at home, and job-sharing. When these programs were made available at the Summit Company, the leader in its industry, only a small percentage of employees participated in them. Rather than adversely affecting our profitability by offering these programs, we should concentrate on offering extensive training that will enable employees to increase their productivity.”

47. The following appeared in a letter to prospective students from the admissions office at Plateau College:

“Every person who earned an advanced degree in science or engineering from Olympus University last year received numerous offers of excellent jobs. Typically, many graduates of Plateau College have gone on to pursue advanced degrees at Olympus. Therefore, enrolling as an undergraduate at Plateau College is a wise choice for students who wish to ensure success in their careers.”

48. The following was excerpted from an article in a farming trade publication:

“Farmers who switched from synthetic to organic farming last year have seen their crop yields decline. Many of these farmers feel that it would be too expensive to resume synthetic farming at this point, given the money that they invested in organic farming supplies and equipment. But their investments will be relatively minor compared to the losses from continued lower crop yields. Organic farmers should switch to synthetic farming rather than persist in an unwise course. And the choice to farm organically is financially unwise, given that it was motivated by environmental rather than economic concerns.”

49. The following appeared as part of an article in a computer magazine:

“A year ago Apex Manufacturing bought its managers computers for their homes and paid for telephone connections so that they could access Apex computers and data files from home after normal business hours. Since last year, productivity at Apex has increased by 15 percent. Other companies can learn from the success at Apex: given home computers and access to company resources, employees will work additional hours at home and thereby increase company profits.”

50. The following appeared in the editorial section of a local paper:

“Applications for advertising spots on KMTV, our local cable television channel, decreased last year. Meanwhile, a neighboring town’s local channel, KOOP, changed its focus to farming issues and reported an increase in advertising applications for the year. To increase applications for its advertisement spots, KMTV should focus its programming on farming issues as well.”

About The Author

gmat essay question examples

Suheb Hussain

Don't limit yourself, don't miss these.

gmat essay question examples

Helpful posts Curated just for you!

gmat essay question examples

You might also like

gmat essay question examples

Browse Related Topics

gmat essay question examples

400+ Practice questions with detailed solutions

10+ hours of ai-driven video lessons, adaptive mock test with esr+ analysis.

www.800score.com

  • What is the GMAT?
  • GMAT Scores and B-Schools
  • How the GMAT CAT Works
  • GMAT Pacing Strategies
  • GMAT Tips & Strategies
  • Reading Intro
  • 1. Passage Classification
  • 2. Analyzing Paragraphs
  • 3. Passage Mapping
  • 4. Find the Main Idea
  • 5. Determine Purpose
  • Detail of the Passage
  • Definition of a Term
  • Support for a Premise
  • Function of Passage Part
  • Cultural Hard Ex. 2
  • Society Passages
  • FREE GMAT PREP COURSE
  • Science Medium Essay 1
  • GMAT Reasoning Intro
  • Most Strongly Supported
  • Argument Structure Questions
  • Ad Hominem Flaws
  • Argumentative Flaws
  • Assumption Questions
  • Parallel Reasoning
  • Parallelism Questions
  • Causal Arguments
  • Statistical Reasoning
  • Weaken Reasoning
  • Sufficient/Necessary
  • Paradox Questions
  • Modify Questions
  • CR Chapter Review
  • Grammar Basics
  • Sentence Correction Tips
  • The Three-Step Method
  • A. Introduction
  • B. Subject / Verb Separation
  • C. Collective Nouns
  • D. Plural / Singular
  • E. Neither / Either
  • F. Or / Nor
  • G. Subject / Verb / Object
  • H. Quantity Words
  • I. Sample Questions
  • B. Adjectives and Adverbs
  • C. Adjectives and Adverbs with Sense Verbs
  • D. Misplaced Modifiers
  • E. Sample Questions
  • B. Lists of Verbs and Parallel Constructions
  • C. Lists of Adjectives or Adverbs
  • D. Comparisons
  • E. Correlative Pairs
  • F. Sample Questions
  • B. Subject vs. Object
  • C. Who vs. Whom
  • D. Singular and Plural Pronouns
  • E. Possessive Pronouns
  • F. Objects of to be verbs
  • G. Relative Pronouns
  • H. Sample Questions
  • B. Verb Tense
  • B. Parallelism
  • C. Comparative and Superlative Forms
  • D. Sample Questions
  • Sample Questions
  • Divisibility
  • Simplifying Rules
  • Exponent Expressions
  • Complex Expressions
  • Inequalities
  • Working With Formulas
  • Intersecting Lines
  • Coordinate Geometry
  • Permutations
  • Problem Variations
  • Combinations
  • Combination or Permutation
  • Simple Probability
  • Independent Events
  • Dependent Events
  • Working Backwards
  • Using Outcomes
  • Other Scenarios
  • Statistics Intro
  • Normal Distribution
  • Chapter Review
  • Introduction
  • Standard Deviation
  • Trick Questions
  • Practice Questions
  • Seven Techniques
  • Ratio and Proportion
  • Distance, Rate, & Time
  • Graphs and Data Interpretation
  • Mean, Median and Mode
  • Integrated Reasoning-Introduction
  • Multi-Source Reasoning
  • Table Analysis
  • Graphics Interpretation
  • Two-Part Analysis
  • AWA Introduction
  • Dissecting Arguments
  • Finding Errors
  • Using Strategy
  • International Students
  • Fill Sentences
  • Qualification
  • Start Strong
  • Active Voice
  • Self-Reference
  • Vague Writing
  • Voice Shifting
  • Colloquialisms
  • Using Hyphens Correctly
  • The Apostrophe
  • Additional Essays
  • 10 Most Common Errors
  • GMAT Classes
  • Mission / About Us

The Real Essay Questions

Home » 800score Free GMAT Prep Course » FREE GMAT PREP COURSE » The Real GMAT Essay Questions

1. Retired Sample Official GMAT Questions

To beat the competition, you will need to do some brainstorming for all 280 AWA questions. Any of them could appear on your GMAT, so you should spend some time preparing in advance. While there are many questions possible, the good news is there are no surprises. You will be able to review all of the potential questions beforehand.

  • 2004_awa_topics. (NOTE: REQUIRES ADOBE ACROBAT 5.0 or better.)

2. Review Sample Essay Answers

1. Number the first ten questions from 1-10 for Analysis of Argument. We have sample essays for those questions.

2. Skim through all of the Analysis of Argument essay questions (Analysis of Issue is not tested on the GMAT). You are guaranteed to see one of these questions on test day, so take quick notes on each of the questions. Then go back and read each one again. Pause for a minute to ponder the topic. At least three or four ideas will probably pop into your mind; jot them down. At this point, don’t try to organize your thoughts or commit to a position.

  • There is no one “correct” response to any AWA question.
  • These essays were written in 30-45 minute periods. They represent 5-6 score essays.

Free COURSE

  • GMAT Scores and B-School
  • Science Medium Ex. 1
  • Science Medium Essay 2
  • C. Comparative / Superlative
  • Number Rules Review
  • Quadratic Equations
  • Algebra Review
  • Geometry Review
  • Statistics Review
  • Using Hyphens
  • Sample GMAT Tests

prep

244 5th Avenue #2638 New York, NY 10001 US Contact Us

GMAT Prep | GMAT Classes GRE Prep | GRE Classes LSAT Prep | LSAT Classes MCAT Classes SAT Classes

© 2023 | Terms

GMAT Sample Essays

All GMAT students know the Argument essay comes first in the test, before we get to the more important Quantitative and Verbal sections, so it’s important not to exhaust ourselves in this first part of the exam. One way to stay fresh for the rest of the exam is to have a good idea of what a high-scoring essay looks like, so we have provided some GMAT sample essays to review. Below we look at a “6” and discuss why it would have likely received a perfect score, then we examine a “4” and discuss how it could have been strengthened. You can find more example essays in the GMAT Official Guide. Happy writing!

ESSAY QUESTION #1:

The following appeared in the editorial section of a national news magazine:

“The rating system for electronic games is similar to the movie rating system in that it provides consumers with a quick reference so that they can determine if the subject matter and contents are appropriate. This electronic game rating system is not working because it is self-regulated and the fines for violating the rating system are nominal. As a result an independent body should oversee the game industry and companies that knowingly violate the rating system should be prohibited from releasing a game for two years.”

Discuss how well reasoned you find this argument. Point out flaws in the argument’s logic and analyze the argument’s underlying assumptions. In addition, evaluate how supporting evidence is used and what evidence might counter the argument’s conclusion. You may also discuss what additional evidence could be used to strengthen the argument or what changes would make the argument more logically sound.

The author concludes that electronic game rating system is not working compared to the movie rating system. He gives reasoning for the argument by stating that electronic companies ability to self manage and regulate the rating system is part of the problem. Author also gives reason for what needs to be done in order for electronic rating system to work, but his reasons are weak. In the next few paragraphs, I will explain why the author’s reasons are weak and what could strengthen the argument.

First, the author mentions that by hiring an independent company to oversee the operation of the electronic rating system would solve the issue. This big assumption that author makes here is that he assumes independent company would do a better job in rating games than electronic game companies themselves. If the electronic gaming companies had a better understanding on how to improve, they might themselves do a better job than independent company to oversee the ratings.

Second, the author mentions that if an electronic company violates the rating system rule, then the penalty would be to prohibit that company from releasing any games for two years. This is another weak point the makes to support his argument because if the electronic company regular product-life cycle is to release each gave every two years, this penalty wouldn’t hurt the company at all. Also, there is not way of assessing how many year of prohibition would be adequate.

Author could have strengthen his argument, if he provided some data point such as from the movie industry to defend this stance that making independent company oversee and prohibition of movie release actually worked in the long-run. Perhaps, some research data that showed making these changes would actually work, would benefit the author’s overall argument stance.

Therefore, the author’s argument that electronic game rating system is not working is weak. Both of the points he made regarding independent company oversee and violation penalty are weak without data showing that it might work.

RECAP: The first thing that stands out in this essay is the organization. The paragraphs as clearly laid out and succinct, and each begin with a great transition word or phrase. The introductory paragraph, while unfortunately uses some unneeded self-reference, clearly demonstrates an understanding of the presented argument, which is mandatory of all “6” essays. Each flaw is then pointed out in a body paragraph, and the author then chooses to include a nice “how to strengthen” paragraph to demonstrate that he/she knows the argument at a more advanced level! The conclusion is clear, and reinforces the claims previously made. While no means perfect, the strong reasoning and clarity of organization definitely give this author a “6”!

ESSAY QUESTION #2:

Political organizations that advocate the use of violence to achieve their goals should be prohibited from operating within our country. Such groups are only interested in achieving short term goals which lead to more serious long term problems.

Political organizations that advocate the use of violence to achieve its goals can sometimes lead to destruction and devastation. However, that claim that such groups are detrimental to society does not follow the same line of reasoning. These groups might be of great help to certain sections of society. Also the claim that short term solutions can only lead to more serious long term problems is stated without any evidence. Hence the above argument is flawed.

Firstly, political organizations might be of great help to certain sections of society. Take for example the Indian National Congress party that helped tons of Indian’s voice their views to the British government during the British rule in india. Although this political organization advocated the use of violence, the organization was critical to India becoming a free of British rule in 1947.

Secondly, the argument assumes that short term solutions lead to more serious long term problems. Short term solutions are often very important in achieving long term goals, irrespective of whether the organizations advocate violence or not. The argument must also state clearly what constitutes violence. In a free country, such as the United States, to prohibit any political organization is to put a hold on their freedom. As long as the violence or aggressive behavior is not illegal, one can not prohibit a political organization from operating.

Thus although the argument seems to convey a valid point, that political organizations that advocate the use of violence should be prohibited from operating, it is worded to strongly and lacks evidence to support its claims that these groups are detrimental to society.

RECAP: The major difference between these two essays is length. More is not always better, but a student who can write 5-6 paragraphs within the given time frame definitely will stand out as a better planner, and as someone who already had a workable template. Furthermore, some minor grammar and spelling mistakes interfere with the essay’s clarity. The thesis, “Hence the above argument is flawed, “ is not as strongly worded as it could be, and doesn’t stand on its own as a powerful statement. The conclusion also seems to weaken the essay by admitting the argument conveys “a valid point.” This concession, without being more specific, weakens the overall thesis. The takeaway: length, specificity, and strength of argument will take this “4” to a “6.”

More AWA Resources: GMAT Essay Writing Guide

Recently viewed courses

Recently viewed.

Find Your Dream School

This site uses various technologies, as described in our Privacy Policy, for personalization, measuring website use/performance, and targeted advertising, which may include storing and sharing information about your site visit with third parties. By continuing to use this website you consent to our Privacy Policy and Terms of Use .

   COVID-19 Update: To help students through this crisis, The Princeton Review will continue our "Enroll with Confidence" refund policies. For full details, please click here.

GMAT Sample Questions

Hidden laptop with hands typing, displaying a test diagram above the keyboard

Want a preview of the question types you'll face on the GMAT? Try your hand at the GMAT practice questions below. Then, check your answers against our in-depth explanations to see how you did.

We pulled these GMAT sample questions from our book Cracking the GMAT and from our test prep course materials. For more GMAT practice, take a full-length practice test with us held under the same testing conditions as the real thing. Find out how you'd score, and get  a personalized score report from us that shows your strengths and weaknesses.

  • GMAT Verbal Questions 
  • GMAT Math Questions
  • GMAT Integrated Reasoning Questions 
  • Essay Prompt 

Below you'll find sample GMAT questions covering the three question types you'll encounter on the Verbal section: Sentence Correction , Critical Reasoning, and Reading Comprehension.

GMAT Sentence Correction Questions

1. In order to better differentiate its product from generic brands, the cereal company first hired a marketing firm that specializes in creating campaigns to build brand awareness and then retools its factory to produce a variety of different shapes of cereal. (A) then retools its factory to produce a variety of different shapes of cereal (B) retools its factory to produce a variety of different shapes of cereal (C) then retooled its factory to produce a variety of different shapes of cereal (D) then will retool its factory to produce a variety of different shapes of cereal (E) then produces a variety of different shapes of cereal through retooling its factory

Answer: (C) The actions of the cereal company are not in parallel form. First the company hired then it retools . Eliminate choice (A). Choice (B) still has the same error. Choice (D) changes the verb form incorrectly to the future tense. Choice (E) rewrites the sentence but retains the error.

[+] See the Answer

2. Believed to be one of the first widely read female authors of the Western world, Christine de Pizan's masterwork The Book of the City of the Ladies , was written in 1405 and is a history of the Western world from the woman's point-of-view. (A) Believed to be one of the first widely read female authors of the Western world (B) Written by one of the first widely read female authors of the Western world (C) One of the first widely read female authors of the Western world, as some believe (D) Written by what some believe as one of the first widely read female authors of the Western world (E) Believed by some as one of the first works by a widely read female author in the Western world

Answer: (B) As written, this sentence has a misplaced modifier error: the book, The Book of the City of the Ladies isn't believed by anyone to be an author— Christine de Pizan is. Choices (A) and (C) repeat that error and can be eliminated. Choices (B) and (D) both change the introductory phrase to clearly refer to a written work, but choice (D) uses the incorrect idiom believe as instead of the correct form, believe to be . Choice (E) repeats that idiom error.

GMAT Critical Reasoning Questions

1. One food writer wrote that reducing the amount of animal products in one's diet can contribute to better health and well-being. Based on this claim, some people are completely eliminating meat from their diets in order to be healthier. The argument above relies on which of the following assumptions?

Answer: (B) The argument states that some people are eliminating meat from their diets because reducing the amount of animal products in one's diet can lead to better health. Meat is only one type of animal product, however. The argument assumes that by eliminating meat, the people are reducing the total amount of animal products in their diets. Choice (A) addresses increasing the amount of vegetables and grains, but the argument just deals with animal products. Choice (B) correctly addresses the people who are eliminating meat and states that those people are not increasing their consumption of dairy, which is another instance of using animal products. Thus, these people are actually reducing the amount of animal products in their diets. Choice (C) addresses most food writers, who are irrelevant to this argument. Choice (D) addresses health lifestyles, which are irrelevant to this particular argument. Choice (E) addresses the reasons behind not eating animal products, which is irrelevant to the argument.

2. Studies reveal that a daily exercise regimen helps stroke survivors regain dexterity in their extremities. Being given an exercise routine and having a consultation with a doctor about the exercise routine have been shown to be effective mechanisms to get patients to exercise daily. From the above information, which of the following statements can be reasonably inferred? (A) A stroke survivor that is given a detailed exercise plan and consults her physician about the plan will regain full dexterity in her extremities. (B) If a stroke survivor is not given an exercise plan and does not consult with a doctor, she will not regain dexterity in her extremities. (C) Stroke survivors who are given an exercise routine and consult with a doctor about that routine will sometimes regain dexterity in their extremities. (D) Being given an exercise routine and having a consultation with a doctor about the routine is the best way to help a stroke survivor regain dexterity in their extremities. (E) Only being given an exercise routine is necessary to regenerate dexterity in the extremities of seniors who have suffered a stroke.

Answer: (C) This is an inference question, so evaluate the passage and then look for an answer choice that can be reasonably inferred from the information. The passage states that a daily exercise regimen helps stroke survivors regain dexterity in their extremities and that survivors who are given an exercise routine and who have a consultation with a doctor about the routine have been shown to be effective at getting patients to exercise daily . So it can be inferred that if a survivor is given a routine and consults with a doctor, they are more likely to exercise daily, which will help them regain dexterity. Choice (A) is an example of extreme language. The phrasing will regain full dexterity is not promised in the information in the passage, as the passage only states that a routine and consultations may help a survivor exercise more. Eliminate (A). Choice (B) is also an example of extreme language. There is no way to discern from the information provided that a strong survivor would not regain dexterity without an exercise routine and a consultation, so eliminate (B). Choice (C) is a reasonable inference to make from the information in the passage so keep (C). Choice (D) also contains the extreme language best way . The information does not compare this method with any other method so eliminate (D). Choice (E) is recycled language and does not address consulting with a doctor so eliminate (E). The correct answer is (C).

GMAT Reading Comprehension Questions

Although oft-maligned in modern culture, the pigeon once stood not only for speed and reliability but also for grace and beauty. Darwin himself became a pigeon fancier after beginning to work with the humble Columbia livia , discovering them to be more fascinating than he had formerly believed. During the Victorian age, in fact, raising show pigeons was a popular hobby, with new breeds continuously arising as amateur (and not-so-amateur) ornithologists crossed animals in the hopes of creating ever more fantastic creatures. One of the most sought-after varieties was known as the Almond Tumbler, a name presumably derived from the color of the birds combined with the distinctive flight style. Over the course of many generations, this bird was so manipulated as to have a beak so small as to prevent the adult birds from feeding their offspring. And yet, it was wildly popular, drawing high prices at auctions and high prizes at competitions. How then did an animal once so well-loved come to be so loathed? As recently as World War II, the military used pigeons to carry messages but today, many people would kick a pigeon before they would feed one. Perhaps it is just a problem of population density - a lack of esteem for that which is ubiquitous. Pigeons have become our constant urban companions and, as such, have been transformed from symbols of peace, plenty, and prosperity, to representatives of disease and decay.

1. The primary purpose of this passage is to (A) convince the reader of the nobility of the pigeon, based on its history as a symbol of virtue (B) dissuade the reader from mistreating a once-majestic animal that has fallen from favor (C) rebut claims that the pigeon carries disease any more frequently than do other domestic animals (D) promote a renewal of pigeon fancying and a resurgence of breeds such as the Almond Tumbler (E) suggest that there might be more to the story of some urban wildlife than is commonly known

Answer: (E) The passage gives a brief description of the pigeon's place in recent human history and then goes on to contrast that with modern perspectives of the birds. Choice (A) goes too far—the author doesn't give any indication of believing the pigeon to be noble. Choice (B) focuses too specifically on a side comment in the second paragraph. Choice (C) also focuses too specifically on a side comments—the passage is not primarily about disease. Choice (D) is too strong—the passage isn't really promoting any specific action. Choice (E) remains neutral and informational, as does the passage.

2. The case of the Almond Tumbler is most analogous to which of the following? (A) a strain of wheat that can be grown in plentiful quantities but loses much of its nutritional value in the process (B) Arabian horses that are able to run at phenomenal speeds due to centuries of careful breeding designed to enhance those physical attributes (C) vitamins that were purported to provide all of the necessary nutrients but have since been found not to be very effective (D) the dachshund, a popular breed of dog that is nonetheless prone to severe back problems, due to weaknesses exacerbated by targeted breeding (E) the wild rock doves that are most commonly found nesting in the faces of cliffs far from human habitation

Answer: (D) The Almond Tumbler is described as a breed of pigeon that was very popular during the Victorian era. The passage also mentions that the selective breeding used to create that particular kind of bird also led to tiny beaks that kept parent birds from feeding their babies. Therefore, the best analogy would be another animal that is popular even though it has problems due to its design. Choice (A) is incorrect because it leaves out the aspect of popularity. Choice (B) is only positive and you need something that's also negative. Choice (C) is not about something that has been bred for a specific purpose, nor does it deal with popularity. Choice (D) correctly refers to a popular animal with a common health problem. Choice (E) does not refer to pigeons that have been bred by humans.

3. The passage suggests that (A) pigeons were once known for flying with celerity (B) the Almond Tumbler was the most beautiful breed of pigeon (C) Darwin was infatuated with his fancy pigeons (D) modern pigeons are dirtier than the fancy pigeons of yore (E) only scientists should breed new kinds of animals

Answer: (A) For a question this open-ended, it's usually best to check each of the answers against the passage. Choice (A) appears to match the opening line of the passage, which states that the pigeon once stood not only for speed and reliability. Choice (B) goes too far—although many Victorians seems to have loved the Tumbler, there's no evidence that it was definitively the most beautiful. Choice (C) also goes too far—the passage mentions that Darwin was fascinated by his pigeons, not that he was infatuated. Choice (D) draws an incorrect assumption—the passage comments that the common opinion has changed, not the pigeon itself. Choice (E) is not supported by the passage, which states that amateurs, as well as trained individuals, bred pigeons.

Below you'll find GMAT sample questions covering the two question types you'll encounter on the Quantitative section: Problem Solving and Data Sufficiency.

Problem Solving Questions

1. A certain company sells tea in loose leaf and bagged form, and in five flavors: Darjeeling, earl grey, chamomile, peppermint, and orange pekoe. The company packages the tea in boxes that contain either 8 ounces of tea of the same flavor and the same form, or 8 ounces of tea of 4 different flavors and the same form. If the order in which the flavors are packed does not matter, how many different types of packages are possible? (A) 12 (B) 15 (C) 20 (D) 25 (E) 30

Answer: (C) Begin by figuring out how many different ways you can package the tea in boxes that contains 8 ounces of tea, all of the same flavor. There are five flavors, each flavor can come in either loose leaf or bagged form, so 5 flavors x 2 forms = 10 different ways to package the tea in boxes that contain only one flavor each. Now find the number of different ways to package 4 different flavors of the same form per box. In this case, you must choose 4 of 5 possible flavors, and order does not matter, so the formula is 5 x 4 x 3 x 2 ⁄ 4 x 3 x 2 x 1 = 5 different ways to combine the 4 flavors. Each combination can come in either loose leaf for bagged form, so you have 2 different forms x 5 different combinations = 10 total possible ways to combine the 4 flavors in either bagged or loose-leaf form. Thus, the total number of combinations is 10 + 10 = 20 total combinations. The answer is choice (C).

2. Karen sold her house at a loss of 25 percent of the price that she originally paid for the house, and then bought another house at a price of 30 percent less than the price she originally paid for her first house. If she sold the first house for $225,000, what was her net gain, in dollars, for the two transactions? (A) $15,000 (B) $25,000 (C) $60,000 (D) $75,000 (E) $90,000

Answer: (A) If Karen sold her first house for $225,000 and at a loss of 25 percent, then 25 percent of the original price equals $225,000. 75 ⁄ 100 x = 225,000, so x, or the price she originally paid, equals $300,000. Thus, Karen lost $75,000 on the sale of her first house. If she bought a second house for a price of 30 percent less than $300,000, then the second house cost $210,000, so she gained $90,0000. $90,000 - $75,000 = $15,000, so the answer is choice (A).

Sample Data Sufficiency Questions

1. In a certain company, at least 200 people own manual transmission vehicles. If 12 percent of the people who own manual transmission vehicles also own automatic transmission vehicles, do more people own automatic transmission vehicles than own manual transmission vehicles? (1) 5 percent of the people who own an automatic transmissions vehicle also own a manual transmission vehicle. (2) 15 people own both an automatic transmission vehicle and a manual transmission vehicle. (A) Statement (1) ALONE is sufficient, but statement (2) alone is not sufficient. (B) Statement (2) ALONE is sufficient, but statement (1) alone is not sufficient. (C) BOTH statements TOGETHER are sufficient, but NEITHER statement ALONE is sufficient. (D) EACH Statement ALONE is sufficient. (E) Statements (1) and (2) TOGETHER are NOT sufficient to answer the question asked, and additional data are needed.

Answer: (A) According to statement (1), 5 percent of the people who own an automatic transmission vehicle also own a manual transmission vehicle. The question also indicates that 12 percent of the people who own a manual transmission vehicle also own an automatic transmission vehicle. Both figures relate to the total number who own both, so that means that 5 percent of the automatic transmission owners = 12 percent of the manual transmission owners. The overlap in ownership makes up a smaller percent of those who own automatic transmission vehicles, so there must be more people who own automatic transmission vehicles. Statement (1) is sufficient, so you can eliminate choices (B), (C), and (E). Statement (2) indicates that 15 people own both an automatic transmission vehicle and a manual transmission vehicle, so you know that 12 percent of the people who own a manual transmission is equal to 15 people. 12 ⁄ 100 = 15, so x = 125. Thus, there are 125 people who own a manual transmission vehicle. However, you have no further information to allow you to calculate the number of people who own automatic transmission vehicles, so statement (2) is insufficient. The answer is choice (A).

2. What is the value of x ⁄ 2 ? (1) x is 1 ⁄ 5 less than 9 ⁄ 10 (2) x is between 2 ⁄ 5 and 4 ⁄ 5 (A) Statement (1) ALONE is sufficient, but statement (2) alone is not sufficient. (B) Statement (2) ALONE is sufficient, but statement (1) alone is not sufficient. (C) BOTH statements TOGETHER are sufficient, but NEITHER statement ALONE is sufficient. (D) EACH Statement ALONE is sufficient. (E) Statements (1) and (2) TOGETHER are NOT sufficient to answer the question asked, and additional data are needed.

Answer: (A) Statement (1) allows you to find the value of x, so you can answer the question. (If x is 1 ⁄ 5 less than 9 ⁄ 10 , then 9 ⁄ 10 - 1 ⁄ 5 = x. 1 ⁄ 5 = 2 ⁄ 10 , so x equals 9 ⁄ 10 - 2 ⁄ 10 = 7 ⁄ 10 . If x equals 7 ⁄ 10 , then x ⁄ 2 = 7 ⁄ 10 divided by 2, or 7 ⁄ 20 .) Statement (1) is sufficient, so eliminate choices (B), (C), and (E). According to statement (2), x is between 2 ⁄ 5 and 4 ⁄ 5 . That means that one possible value for x is 3 ⁄ 5 , but another possible value is 7 ⁄ 10 . Statement (2) is insufficient, so the answer is choice (A).

Below you'll find examples of how you'll be asked to use a chart, graph, or table to answer questions on the Integrated Reasoning section.

Sample Integrated Reasoning Questions

Item 1: Andre is buying gifts for his office staff. He wants to spend exactly $280 and he can buy either sweatshirts, which cost $22, or baseball caps, which cost $26. In the table below, choose the number of sweatshirts and the number of baseball caps that Andre should buy.

Answer: Sweatshirts, 8; Baseball caps 4 To solve this question, systematically test out the answer choices. The equation you need to solve is 22s + 26h = 280, in which both s and h are integers and s represents the number of sweatshirts and h represents the number of baseball caps. So, start with plugging in 4 for sweatshirts and see if the number of baseball caps is an integer. 22(4) +26h = 280 h = 7.38 Since the number of baseball caps is not an integer, Andre could not have bought 4 sweatshirts. Keep trying more sweatshirts one by one until you find an answer that will you an integer value for baseball caps. 8 sweatshirts will give you 4 baseball caps.

GMAT sample question

Question 2-1 The ratio of the U.S. population in 2000 to the U.S. population in 1900 is closest to __. (A) 1 to 4 (B) 2 to 7 (C) 2 to 1 (D) 3 to 1 (E) 11 to 3

Answer: (E, 11 to 3) According to the graph, the U.S. population in 2000 was a little bit more than 275 million, and the U.S. population in 1900 was a little over 75 million. Since the question asks what the ratio is "closest to," these numbers are good enough to approximate. 275 to 75 can be reduced by 5 to get 55 to 15, which can be reduced by 5 again to get 11 to 3. Alternatively, you could reduce 275 to 75 by 25 to get this same ratio.

Question 2-2 The U.S. population in 1950 was approximately __ of the U.S. population in 1850. (A) 800% (B) 600% (C) 200% (D) 85% (E) 15%

Answer: (B, 600%) The question asks what percent the U.S. population in 1950 is of the U.S. population in 1850. To get this you need to calculate population 1950 ⁄ population 1850 x 100. Since the U.S. population in 1950 is higher, you want something that is greater than 100%. Eliminate 85% and 15%. Since the sentence says "approximate" and also since the remaining answer choices are not close to each other, you can estimate the values. According to the chart, the population in 1950 was about 150 million and the population in 1850 was about 25 million. Therefore, you need to calculate 150 ⁄ 25 x 100 = 6 x 100 = 600%.

Question 2-3 The U.S. population increased by approximately __ from 1900 to 1950. (A) 25% (B) 33% (C) 50% (D) 100% (E) 200%

Answer: (D, 100%) To get percent increase, you need to use the formula difference ⁄ original x 100. The population in 1900 was about 75 million, and the population in 1950 was about 150 million. The difference between the two figures is 75 million. Therefore, the percent increase is 75 ⁄ 75 x 100 = 100%.

Below you'll find a sample Analytical Writing Assessment (AWA) question. On the GMAT you'll have 30 minutes to write a critique of the argument.

Analysis of an Argument

The following appeared as part of a medical advertisement in a magazine.

A new medical test that allows the early detection of a particular disease will prevent the deaths of people all over the world who would otherwise die from the disease. The test has been extremely effective in allowing doctors to diagnose the disease six months to a year before it would have been spotted by conventional means.

Discuss how logically convincing you find this argument. In explaining your point of view, be sure to evaluate the line of reasoning and the use of evidence in the argument. For example, it may be necessary to consider what questionable assumptions underlie the thinking and what other explanations or counterexamples might weaken the arguments conclusion. You can also discuss what kind of evidence would strengthen or refute the argument, what changes in the argument would make it more logically persuasive, and what, if anything, would enable you to better evaluate its conclusion.

How will you score?

Take a GMAT practice test with us under the same conditions as the real thing. You'll get a personalized score report highlighting your strengths and areas of improvement.

START A FREE PRACTICE TEST

  • GMAT  

Featured Business Schools For You

Find MBA Programs Matched to Your Interests

Explore our featured business schools to find those that are looking for students like you.

Best Online MBA seal

Top Online MBA Programs

On a mission to increase your salary? Our Top 50 Online MBA ranking is based on academics, career outcomes, tech platforms, and more.

Best Career Prospects

Best Career Prospects

Find out which schools have the best track records for getting students jobs—and the highest starting salaries.

Top Entrepreneurship 2023 seal

Top Schools for Entrepreneurship

Ready to build your own business from the ground up? Check out these 50 graduate programs.

What would you score on the MCAT today?

Thank you! Look for the MCAT Review Guide in your inbox.

I already know my score.

Enrollment Advisor

1-800-2REVIEW (800-273-8439) ext. 1

1-877-LEARN-30

Mon-Fri 9AM-10PM ET

Sat-Sun 9AM-8PM ET

Student Support

1-800-2REVIEW (800-273-8439) ext. 2

Mon-Fri 9AM-9PM ET

Sat-Sun 8:30AM-5PM ET

Partnerships

  • Teach or Tutor for Us

College Readiness

International

Advertising

Affiliate/Other

  • Enrollment Terms & Conditions
  • Accessibility
  • Cigna Medical Transparency in Coverage

Register Book

Local Offices: Mon-Fri 9AM-6PM

  • SAT Subject Tests

Academic Subjects

  • Social Studies

Find the Right College

  • College Rankings
  • College Advice
  • Applying to College
  • Financial Aid

School & District Partnerships

  • Professional Development
  • Advice Articles
  • Private Tutoring
  • Mobile Apps
  • Local Offices
  • International Offices
  • Work for Us
  • Affiliate Program
  • Partner with Us
  • Advertise with Us
  • International Partnerships
  • Our Guarantees
  • Accessibility – Canada

Privacy Policy | CA Privacy Notice | Do Not Sell or Share My Personal Information | Your Opt-Out Rights | Terms of Use | Site Map

©2024 TPR Education IP Holdings, LLC. All Rights Reserved. The Princeton Review is not affiliated with Princeton University

TPR Education, LLC (doing business as “The Princeton Review”) is controlled by Primavera Holdings Limited, a firm owned by Chinese nationals with a principal place of business in Hong Kong, China.

gmat essay question examples

How to write your GMAT AWA Essay [Effectively]

gmat essay question examples

INTRODUCTION

Hello there!

If you’ve found your way to this blog, we assume that you have begun to think about that often-neglected section of the GMAT – the AWA!

If wishes could come true, we’re guessing that GMAT aspirants would have wished away the AWA section away by now!

The AWA involves staring at a blank page and cursor and thinking up stuff to write, not an easy task for many people.

It also consumes a lot of mental bandwidth at the very beginning of the test.

To top it off, it does not even add to your final score!

We can understand why it isn’t your favorite section on the GMAT 🙂

But here’s the good news! There are not many shortcuts to mastering GMAT Quant and Verbal, but there are shortcuts to cracking the AWA section.

By the time you finish reading this guide, you will know what these hacks are. You will learn how to write an effective AWA essay that gets you a good score and leaves you charged for the real test that lies ahead.

This blog will teach you –

  • What you need to know BEFORE you start preparing for the AWA
  • How to use the 30 minutes allotted to AWA to maximum effect
  • How to use a template to make the AWA writing process simpler

Besides this, you will find 8 sample AWA essays to observe and learn from.

Happy reading! 🙂

7 Things to remember before you start prepping for the AWA

gmat essay question examples

Before you dive into AWA preparation, there are a few things you should know about the AWA. Many of these facts will ease your AWA fears and bring a smile to your face!

1. Why is the AWA section on the GMAT?

Each section of the GMAT is carefully constructed ( at the expense of millions of dollars, we kid you not!), to test your readiness for an MBA program and for your post-MBA career. One of the skills you will definitely need post-MBA is an ability to analyze an argument impartially and convey your perspective clearly.

This is what the AWA tests you on.

2. On the AWA, you need to be a lawyer, that is, you need to find faults with the given argument. You do not need to be a journalist, that is, you do not need to write about all possible perspectives of an issue.

Also remember, that this is an analysis, not an opinion piece. Do not bring YOUR perspective and your opinions into the essay. Your only goal is to analyse the given argument.

3. Unlike the Quant and Verbal sections, where your thumb-rule should be to get as high a score as possible, we’d suggest that you do not expend too much mental energy on the AWA, trying to score a 6 on 6.

Getting a perfect 6 will look good on your GMAT score card, and will sound great as you’re telling your friends about it. But it will not be the make-or-break factor in your application. A 4 or a 5 is good enough.

4. The AWA is graded by an E-reader application and by a human reader. Since there is an element of automated grading, you can rig the test to an extent. We’ve found that if you write a substantial essay of over 500 words, and if you structure the argument well ( check our CrackVerbal template in the following chapter), you are almost guaranteed to get a 4+ score!

5. Assuming that you prepare for the GMAT over a course of three months, we recommend that you practice writing 5 to 10 essays, and make sure you get feedback for all of them. If you cover this much practice ground, you’re good to go!

6. One of the best things about the AWA section is that you know all of the questions beforehand (yes, they’re all up there on the GMAC site – Analytical Writing Section ). So you do not have to go hunting for ‘authentic’ AWA essay questions.

7. Now you can choose the order in which you want to take up the sections before starting the test. It is advisable to keep in mind the order that would be helpful for you and prepare for the AWA based on that strategy. This is a recent change to the GMAT test structure. It was introduced in July 2017. We have done a detailed analysis of what this means to an Indian GMAT test-taker in the this blog

8. We saved the coolest point for last 🙂

The AWA lends itself very easily to the use of an essay template. No matter what the argument prompt is, you can bet that there will be at least 3 glaring errors of logic in it. You can, therefore, use a template to structure your AWA essay. Using a template takes most of the stress away from the AWA section.

In the few minutes before you start, you can jot down the template on your scratchpad, so that you don’t have to remember it anymore. Also, because you can plan many of your sentences beforehand, you can get at least a 100 words down before you even read the question!

There are a lot of templates on the internet – probably the most famous one being the Chineseburned AWA template.

At CrackVerbal, we have our own template for the AWA, a modified version of the Chineseburned template. We call it the CrackVerbal AWA Template on Steroids! 🙂

The AWA Writing Process

MBA Essay Writing

1. Write your templatized response

This should take you about 5 minutes:

Type out your prepared template response. Below is a sample. We definitely do not recommend that you use the same words. What you can do, however, is read a few templates on the net, and then write your own. Since you have written it yourself, it will be that much easier to memorise it.

CRACKVERBAL AWA TEMPLATE

The argument claims that < restate the argument >. Stated in this way the argument fails to take into account a few key factors which could call the conclusion to question. It rests on some assumptions, for which there is no clear evidence. Therefore, the argument is unconvincing and falls apart at the seams.

Paragraph 1:

1. Firstly, ( ) 2. This statement is a stretch and not substantiated in any way. 3. The argument would have been much clearer if ( )

Paragraph 2:

1. Second ( ). This is again a very weak and unsupported claim as the argument ( ). 2. For example, 3. This argument would have sounded a lot more convincing if 4. In addition, it would have been strengthened ever further if the argument provided evidence that

Paragraph 3:

1. Finally, the argument concludes that 2. However, what is not clear here is ( ) 3. If there had been evidence to support ( )

In summary, the argument fails to convince because of the faulty assumptions aforementioned. If the argument had drawn upon examples as suggested, and thereby plugged in the holes in the reasoning, it would have been far sounder on the whole.

2. Brainstorm

Now that you have put the pre-planned portion of the essay down, it’s time to read the AWA prompt and wear your thinking hat. GMAT, in its politically correct, non-partisan way, says ‘Discuss how well-reasoned you find this argument’. Remember however, that an AWA argument is never well-reasoned!

There are always a couple of glaring flaws in logic you can pounce on. If these flaws do not occur to you immediately, because of test-day stress, do not assume that you have been given a particularly sound argument. There is no such thing on the AWA!

If you’re unable to be critical, imagine that the author of the argument is somebody you dislike..a teacher you hated at college, or that guy who overtook you and almost dented your car this morning! There, now you’re in the right frame of mind to attack the argument 🙂

Before you do so, you need to understand the three elements of the argument – Conclusion, Premise and Assumptions.

Let us look at an example, and detect these three elements.

“Most companies would agree that as the risk of physical injury occurring on the job increases, the wages paid to employees should also increase. Hence it makes financial sense for employers to make the workplace safer: they could thus reduce their payroll expenses and save money.”

The conclusion is the decision/statement that the author has arrived at. In this case, the conclusion is the last sentence – “Hence it makes financial sense for employers to make the workplace safer: they could thus reduce their payroll expenses and save money.”

The premises are the building blocks of facts on which the conclusion rests. In other words, a premise is what is offered as support for the conclusion. In this case, the premise is – Most companies would agree that as the risk of physical injury occurring on the job increases, the wages paid to employees should also increase.

Assumptions are the unstated, unwritten premises that plug the gap between the written premises and the conclusion. It is the assumptions that you need to attack on the AWA!

How do you attack assumptions?

Remember that an assumption can be incorrect for a variety of reasons. Here are a few types of incorrect assumptions –

1. The Sampling Assumption – The sampling argument assumes that a small group is representative of a much larger group to which it belongs.

2. The illogical analogy assumption – The illogical analogy states that because something applies to A, it applies to B also.

3. The Causal Assumption – The Causal Assumption confuses correlation with causation. That means, just because ‘A’ usually occurs after ‘B’ occurs, does not necessarily imply that B happens because of A.

4. The Data Bias – This occurs when the data for a statistical inference itself is drawn from a sample that is not representative of the population under consideration. This is a case of faulty data leading to faulty assumptions.

5. The Non Sequitur – This simply means, finding a connection where there is none. Non Sequitur means “does not follow,” which is short for: the conclusion does not follow from the premise.

Don’t let these categories overwhelm you. We’ve put them down here to get you thinking. However, you can find faulty assumptions with ease, even if you have no clue what a non sequitur is!

As you brainstorm, you will need to jot down your thoughts on the scratchpad. Keep it crisp and brief. Make sure you have these things down –

1. Conclusion + Premise: 2. Flawed Assumption #1: 3. Flawed Assumption #2: 4. Flawed Assumption #3:

For each assumption, also make a cursory note of why it is flawed, an example that talks about why it is flawed, and what additional data would strengthen the argument ( or if you are convinced that you can remember these additional details without having to make a note of them, you can get on with the writing! )

This should take you about 15 minutes:

Here is where you fill in your templatized response with specific details.

The only detail you need to add to the first paragraph is a summary of the argument that is presented. In the above template, your summary should go here ->

1. Start off by pointing out the first flawed assumption. 2. Explain why this assumption is flawed. 3. Give an example that supports the flaw. 4. Explain what further information could have strengthened this argument.

1. Start off by pointing out the second flawed assumption. 2. Explain why this assumption is flawed. 3. Give an example that supports the flaw. 4. Explain what further information could have strengthened this argument.

Paragraph 4:

1. Start off by pointing out the third flawed assumption. 2. Explain why this assumption is flawed. 3. Give an example that supports the flaw. 4. Explain what further information could have strengthened this argument.

Paragraph 5:

This is the concluding paragraph. You already have it down in your template! 🙂

4. Proofread

Are you wondering if three minutes is really enough time to proof-read a 500 word essay?

Here’s the deal – The AWA section is about whether you can analyse an argument and discuss it in an articulate manner. It is not a test of grammar and spelling. Hence, the GMAT will excuse minor errors in spelling and grammar.

However, you should understand that a human reader is going to be reviewing your work, and any human reader will have an unconscious bias against bad grammar and spellings. Hence, you want to keep your essay as error-free as possible, without worrying about it too much.

Three minutes should be able time for you to quickly glance through the document and make sure you haven’t made any obvious errors.

Voila! 🙂 Your AWA essay is ready!

Also Read: GMAT Section Selection – Everything you need to know

Sample AWA Essays

gmat essay question examples

Sample Essay 1

”Most companies would agree that as the risk of physical injury occurring on the job increases, the wages paid to employees should also increase. Hence it makes financial sense for employers to make the workplace safer: they could thus reduce their payroll expenses and save money.”

Discuss how well reasoned you find this argument. In your discussion be sure to analyze the line of reasoning and the use of evidence in the argument.

For example, you may need to consider what questionable assumptions underlying the thinking and what alternative explanations or counter examples might weaken the conclusion. You can also discuss what sort of evidence would strengthen or refute the argument, what changes in the argument would make it more logically sound, and what, if anything, would help you better evaluate its conclusion.

Introduction:

This argument states that it makes financial sense for employers to make the workplace safer because lower wages could then be paid to employees. This conclusion is based on the premise that as the risk of physical injury increases, the wages paid to employees should also increase. However, this argument makes several unsupported assumptions. For example, the argument assumes that the costs associated with making the workplace safe do not outweigh the increased payroll expenses due to hazardous conditions.

Body Paragraph 1

The first issue to be addressed is whether increased labor costs justify large capital expenditures to improve the work environment. Clearly one could argue that if making the workplace safe would cost an exorbitant amount of money in comparison to leaving the workplace as is and paying slightly increased wages than it would not make sense to improve the work environment. For example, if making the workplace safe would cost $100 million versus additional payroll expenses of only $5,000 per year, it would make financial sense to simply pay the increased wages. No business or business owner would pay all that extra money just to save a couple dollars and improve employee health and relations. To consider this, a cost benefit analysis must be made. I also feel that although a cost benefit analysis should be the determining factor with regard to these decisions making financial sense, it may not be the determining factor with regard to making social, moral and ethical sense.

Body Paragraph 2

Finally one must understand that not all work environments can be made safer. For example, in the case of coal mining, a company only has limited ways of making the work environment safe. While companies may be able to ensure some safety precautions, they may not be able to provide all the safety measures necessary. In other words, a mining company has limited ability to control the air quality within a coal mine and therefore it cannot control the risk of employees getting black. In other words, regardless of the intent of the company, some jobs are simply dangerous in nature.

In conclusion, while at first it may seem to make financial sense to improve the safety of the work environment sometimes it truly does not make financial sense. Furthermore, financial sense may not be the only issue a company faces. Other types of analyses must be made such as the social ramifications of an unsafe work environment and the overall ability of a company to improve that environment (i.e., coal mine). Before any decision is made, all this things must be considered, not simply the reduction of payroll expenses.

Sample Essay 2

The following appeared in a memorandum issued by a large city’s council on the arts.

“In a recent citywide poll, fifteen percent more residents said that they watch television programs about the visual arts than was the case in a poll conducted five years ago. During these past five years, the number of people visiting our city’s art museums has increased by a similar percentage. Since the corporate funding that supports public television, where most of the visual arts programs appear, is now being threatened with severe cuts, we can expect that attendance at our city’s art museums will also start to decrease. Thus some of the city’s funds for supporting the arts should be reallocated to public television.”

Discuss how well reasoned… etc.

Introduction

In this argument the author concludes that the city should allocate some of its arts funding to public television. The conclusion is based on two facts: (1) attendance at the city’s art museum has increased proportionally with the increases in visual-arts program viewing on public television, and (2) public television is being threatened by severe cuts in corporate funding. While this argument is somewhat convincing, a few concerns need to be addressed.

To begin with, the argument depends on the assumption that increased exposure to the visual arts on television, mainly public television, has caused a similar increase in local art-museum attendance. However, just because increased art-museum attendance can be statistically correlated with similar increases in television viewing of visual-arts programs, this does not necessarily mean that the increased television viewing of arts is the cause of the rise in museum attendance.

Moreover, perhaps there are other factors relevant to increased interest in the local art museum; for instance, maybe a new director had procured more interesting, exciting acquisitions and exhibits during the period when museum attendance increased, in addition, the author could be overlooking a common cause of both increases. It is possible that some larger social or cultural phenomenon is responsible for greater public interest in both television arts programming and municipal art museums.

Body Paragraph 3

To be fair, however, we must recognize that the author’s assumption is a special case of a more general one that television viewing affects people’s attitudes and behavior. Common sense and observation tell me that this is indeed the case. After all, advertisers spend billions of dollars on television ad time because they trust this assumption as well.

In conclusion, I am somewhat persuaded by this author’s line of reasoning. The argument would be strengthened if the author were to consider and rule out other significant factors that might have caused the increase in visits to the local art museum.

Sample Essay 3

The following appeared in a report presented for discussion at a meeting of the directors of a company that manufactures parts for heavy machinery.

“The falling revenues that the company is experiencing coincide with delays in manufacturing. These delays, in turn, are due in large part to poor planning in purchasing metals. Consider further that the manager of the department that handles purchasing of raw materials has an excellent background in general business, psychology, and sociology, but knows little about the properties of metals. The company should, therefore, move the purchasing manager to the sales department and bring in a scientist from the research division to be manager of the purchasing department.”

In response to a coincidence between falling revenues and delays in manufacturing, the report recommends replacing the manager of the purchasing department. The grounds for this action are twofold. First, the delays are traced to poor planning in purchasing metals. Second, the purchasing manager’s lack of knowledge of the properties of metals is thought to be the cause of the poor planning. It is further recommended that the position of the purchasing manager be filled by a scientist from the research division and that the current purchasing manager be reassigned to the sales department. In support of this latter recommendation, the report states that the current purchasing manager’s background in general business, psychology, and sociology equip him for this new assignment. The recommendations advanced in the report are questionable for two reasons.

To begin with, the report fails to establish a causal connection between the falling revenues of the company and the delays in manufacturing. The mere fact that falling revenues coincide with delays in manufacturing is insufficient to conclude that the delays caused the decline in revenue. Without compelling evidence to support the causal connection between these two events, the report’s recommendations are not worthy of consideration.

Second, a central assumption of the report is that knowledge of the properties of metals is necessary for planning in purchasing metals. No evidence is stated in the report to support this crucial assumption. Moreover, it is not obvious that such knowledge would be required to perform this task. Since planning is essentially a logistical function, it is doubtful that in-depth knowledge of the properties of metals would be helpful in accomplishing this task.

In conclusion, this is a weak argument. To strengthen the recommendation that the manager of the purchasing department be replaced, the author would have to demonstrate that the falling revenues were a result of the delays in manufacturing. Additionally, the author would have to show that knowledge of the properties of metals is a prerequisite for planning in purchasing metals.

Sample Essay 4

The following appeared in an announcement issued by the publisher of The Mercury, a weekly newspaper.

“Since a competing lower-priced newspaper, The Bugle, was started five years ago, The Mercury’s circulation has declined by 10,000 readers. The best way to get more people to read The Mercury is to reduce its price below that of The Bugle, at least until circulation increases to former levels. The increased circulation of The Mercury will attract more businesses to buy advertising space in the paper.”

A newspaper publisher is recommending that the price of its paper, The Mercury, be reduced below the price of a competing newspaper, The Bugle. This recommendation responds to a severe decline in circulation of The Mercury during the 5-year period following the introduction of The Bugle. The publisher’s line of reasoning is that lowering the price of The Mercury will increase its readership, thereby increasing profits because a wider readership attracts more advertisers. This line of reasoning is problematic in two critical respects.

While it is clear that increased circulation would make the paper more attractive to potential advertisers, it is not obvious that lowering the subscription price is the most effective way to gain new readers. The publisher assumes that price is the only factor that caused the decline in readership. But no evidence is given to support this claim. Moreover, given that The Mercury was the established local paper, it is unlikely that such a mass exodus of its readers would be explained by subscription price alone.

There are many other factors that might account for a decline in The Mercury’s popularity. For instance, readers might be displeased with the extent and accuracy of its news reporting, or the balance of local to other news coverage. Moreover, it is possible The Mercury has recently changed editors, giving the paper a locally unpopular political perspective. Or perhaps readers are unhappy with the paper’s format, the timeliness of its feature articles, its comics or advice columns, the extent and accuracy of its local event calendar, or its rate of errors.

In conclusion, this argument is weak because it depends on an oversimplified assumption about the causal connection between the price of the paper and its popularity. To strengthen the argument, the author must identify and explore relevant factors beyond cost before concluding that lowering subscription prices will increase circulation and, thereby, increase advertising revenues.

Sample Essay 5

The following appeared as part of an article in a magazine devoted to regional life.

“Corporations should look to the city of Helios when seeking new business opportunities or a new location. Even in the recent recession, Helios’s unemployment rate was lower than the regional average. It is the industrial center of the region, and historically it has provided more than its share of the region’s manufacturing jobs. In addition, Helios is attempting to expand its economic base by attracting companies that focus on research and development of innovative technologies.”

In this argument corporations are urged to consider the city of Helios when seeking a new location or new business opportunities. To support this recommendation, the author points out that Helios is the industrial center of the region, providing most of the region’s manufacturing jobs and enjoying a lower-than-average unemployment rate. Moreover, it is argued, efforts are currently underway to expand the economic base of the city by attracting companies that focus on research and development of innovative technologies. This argument is problematic for two reasons.

To begin with, it is questionable whether the available labor pool in Helios could support all types of corporations. Given that Helios has attracted mainly industrial and manufacturing companies in the past, it is unlikely that the local pool of prospective employees would be suitable for corporations of other types. For example, the needs of research and development companies would not be met by a labor force trained in manufacturing skills. For this reason, it’s unlikely that Helios will be successful in its attempt to attract companies that focus or research and development of innovative technologies.

Another problem with the available work force is its size. Due to the lower than average unemployment rate in Helios, corporations that require large numbers of workers would not find Helios attractive. The fact that few persons are out of work suggests that new corporations will have to either attract new workers to Helios or pay the existing workers higher wages in order to lure them away from their current jobs. Neither of these alternatives seems enticing to companies seeking to relocate.

In conclusion, the author has not succeeded in providing compelling reasons for selecting Helios as the site for a company wishing to relocate. In fact, the reasons offered function better as reasons for not relocating to Helios. Nor has the author provided compelling reasons for companies seeking new business opportunities to choose Helios.

Sample Essay 6

The following appeared in the health section of a magazine on trends and lifestyles.

“People who use the artificial sweetener aspartame are better off consuming sugar, since aspartame can actually contribute to weight gain rather than weight loss. For example, high levels of aspartame have been shown to trigger a craving for food bydepleting the brain of a chemical that registers satiety, or the sense of being full. Furthermore, studies suggest that sugars, if consumed after at least 45 minutes of continuous exercise, actually enhance the body’s ability to burn fat. Consequently, those who drink aspartame-sweetened juices after exercise will also lose this calorie-burning benefit. Thus it appears that people consuming aspartame rather than sugar are unlikely to achieve their dietary goals.”

In this argument the author concludes that people trying to lose weight are better off consuming sugar than the artificial sweetener aspartame. To support this conclusion the author argues that aspartame can cause weight gain by triggering food cravings, whereas sugar actually enhances the body’s ability to burn fat. Neither of these reasons provides sufficient support for the conclusion.

The first reason that aspartame encourages food cravings is supported by research findings that high levels of aspartame deplete the brain chemical responsible for registering a sense of being satedHidden text (sated, sating ), or full. But the author’s generalization based on this research is unreliable. The research was based on a sample in which large amounts of aspartame were administered; however, the author applies the research findings to a target population that includes all aspartame users, many of whom would probably not consume high levels of the artificial sweetener.

The second reason that sugar enhances the body’s ability to burn fat is based on the studies in which experimental groups, whose members consumed sugar after at least 45 minutes of continuous exercise, showed increased rates of fat burning. The author’s general claim, however, applies to all dieters who use sugar instead of aspartame, not just to those who use sugar after long periods of exercise. Once again, the author’s generalization is unreliable because it is based on a sample that clearly does not represent all dieters.

To conclude, each of the studies cited by the author bases its findings on evidence that does not represent dieters in general; for this reason, neither premise of this argument is a reliable generalization. Consequently, I am not convinced that dieters are better off consuming sugar instead of aspartame.

Sample Essay 7

The following appeared in the editorial section of a corporate newsletter.

“The common notion that workers are generally apathetic about management issues is false, or at least outdated: a recently published survey indicates that 79 percent of the nearly 1,200 workers who responded to survey questionnaires expressed a high level of interest in the topics of corporate restructuring and redesign of benefits programs.”

Based upon a survey among workers that indicates a high level of interest in the topics of corporate restructuring and redesign of benefits programs, the author concludes that workers are not apathetic about management issues. Specifically, it is argued that since 79 percent of the 1200 workers who responded to survey expressed interest in these topics, the notion that workers are apathetic about management issues is incorrect. The reasoning in this argument is problematic in several respects.

First, the statistics cited in the editorial may be misleading because the total number of workers employed by the corporation is not specified. For example, if the corporation employs 2000 workers, the fact that 79 percent of the nearly 1200 respondents showed interest in these topics provides strong support for the conclusion. On the other hand, if the corporation employs 200,000 workers, the conclusion is much weaker.

Another problem with the argument is that the respondents’ views are not necessarily representative of the views of the work force in general. For example, because the survey has to do with apathy, it makes sense that only less apathetic workers would respond to it, thereby distorting the overall picture of apathy among the work force. Without knowing how the survey was conducted, it is impossible to assess whether or not this is the case.

A third problem with the argument is that it makes a hasty generalization about the types of issues workers are interested in. It accords with common sense that workers would be interested in corporate restructuring and redesign of benefits programs, since these issues affect workers very directly. However, it is unfair to assume that workers would be similarly interested in other management issues—ones that do not affect them or affect them less directly.

In conclusion, this argument is not convincing as it stands. To strengthen it, the author would have to show that the respondents account for a significant and representative portion of all workers. Additionally, the author must provide evidence of workers’ interest other management topics—not just those that affect workers directly.

Sample Essay 8

The following appeared in the opinion column of a financial magazine.

“On average, middle-aged consumers devote 39 percent of their retail expenditure to department store products and services, while for younger consumers the average is only 25 percent. Since the number of middle-aged people will increase dramatically within the next decade, department stores can expect retail sales to increase significantly during that period. Furthermore, to take advantage of the trend, these stores should begin to replace some of those products intended to attract the younger consumer with products intended to attract the middle-aged consumer.”

Based on an expected increase in the number of middle-aged people during the next decade, the author predicts that retail sales at department stores will increase significantly over the next ten years. To bolster this prediction, the author cites statistics showing that middle-aged people devote a much higher percentage of their retail expenditure to department-store services and products than younger consumers do. Since the number of middle-aged consumers is on the rise and since they spend more than younger people on department-store goods and services, the author further recommends that department stores begin to adjust their inventories to capitalize on this trend. Specifically, it is recommended that department stores increase their inventory of products aimed at middle- aged consumers and decrease their inventory of products aimed at younger consumers. This argument is problematic for two reasons.

First, an increase in the number of middle-aged people does not necessarily portend an overall increase in department-store sales. It does so only on the assumption that other population groups will remain relatively constant. For example, if the expected increase in the number of middle-aged people is offset by an equally significant decrease in the number of younger people, there will be little or no net gain in sales.

Second, in recommending that department stores replace products intended to attract younger consumers with products more suitable to middle-aged consumers, the author assumes that the number of younger consumers will not also increase. Since a sizable increase in the population of younger consumers could conceivably offset the difference in the retail expenditure patterns of younger and middle- aged consumers, it would be unwise to make the recommended inventory adjustment lacking evidence to support this assumption.

This argument is unacceptable. To strengthen the argument the author would have to provide evidence that the population of younger consumers will remain relatively constant over the next decade.

We hope that our strategies help you conquer GMAT AWA with enough and more energy to spare for the sections that follow!

Now that you’ve figured out how to tackle the AWA section, do you want to put theory to practice and get your AWA essay graded?

Our experts here at CrackVerbal will evaluate and grade your AWA essay and give you specific, actionable feedback.

gmat essay question examples

GMAT essay topics: Expert tips for a high-scoring performance

GMAT essay topics: Expert tips for a high-scoring performance

The GMAT essay, also known as the Analytical Writing Assessment (AWA), is a section of the GMAT exam that can often be intimidating for test-takers. However, with the right preparation and understanding of the essay topics, test-takers can excel and achieve a high score.

The AWA section is designed to evaluate the test-taker's ability to analyse arguments, express ideas clearly and concisely, and support their viewpoints with evidence. This section consists of two types of essay prompts, an Analysis of an Issue and an Analysis of an Argument, each of which requires specific skills and strategies.

In this article, we will provide expert examples of both types of GMAT essay topics and tips on how to approach them to help you achieve a high-scoring performance in the AWA section.

Table of Contents

1. analysis of an argument, 2. analysis of an issue, gmat essay topics for analysis of an argument, gmat essay topics for analysis of an issue, gmat essay practice questions, 1. gmat essay topics for analysis of an argument, 2. gmat essay topics for analysis of an issue, 1. understand the prompt and task, 2. plan your essay, 3. analyse the argument or issue, 4. use specific examples, 5. write clearly and concisely, 6. follow the standard essay format, 7. practice, practice, practice, what is the gmat essay, how is the gmat essay scored, how do i prepare for the gmat essay.

The GMAT essay, also known as the Analytical Writing Assessment (AWA), is a section of the GMAT exam that requires you to write two essays within a total of 60 minutes.

There are two types of essay prompts in the GMAT AWA section:

  • Analysis of an Argument
  • Analysis of an Issue

In this type of essay prompt, you will be given an argument that you need to analyse and critique. You will be required to identify the assumptions that the argument makes and evaluate the evidence that it provides. You will need to explain why the argument is flawed or weak and suggest ways to improve it.

In this type of essay prompt, you will be given a statement or question related to a social or political issue, and you will be asked to take a stance on the issue and support your position with evidence and examples. You will need to provide a clear argument and present evidence that supports your viewpoint.

Also read: 5 IELTS Sample Essays

GMAT Analytical Writing Section

The GMAT Analytical Writing Assessment section assesses your ability to analyse arguments and communicate your thoughts effectively in written form. In this section, you are presented with an argument or an issue prompt and are required to critically evaluate and provide a well-structured response within a given time frame.

Although the AWA section is not as heavily weighted as the Quantitative and Verbal sections, it still plays a role in showcasing your analytical abilities and written communication skills to business schools. A well-written essay in the AWA section can leave a positive impression on the admissions committee and contribute to your overall application.

  • The construction of a new highway through the rural town of Pineville is essential to improve connectivity and drive economic growth. Discuss the validity of this argument and evaluate its potential impact on the community.
  • The introduction of a mandatory recycling program in our city is necessary to combat the growing environmental concerns and promote sustainable practices. Analyse the reasoning behind this argument and assess its potential effectiveness.
  • Expanding the public library's operating hours in the urban neighbourhood of Riverton will provide greater access to educational resources and foster a culture of lifelong learning. Evaluate the logic behind this proposal and its potential benefits for the community.
  • Establishing stricter regulations on the use of single-use plastics is crucial to mitigate the environmental damage caused by plastic pollution. Assess the reasoning behind this argument and discuss its potential impact on businesses and consumers.
  • The implementation of a tax incentive program to encourage the adoption of solar energy systems in residential areas will promote renewable energy usage and reduce carbon emissions. Evaluate the logic behind this proposal and its potential long-term effects.
  • Constructing a new sports stadium in the city centre will not only revitalise the local economy but also enhance the city's reputation as a sports destination. Analyse the soundness of this argument and its potential implications for the community.
  • Implementing stricter gun control measures is necessary to address the rising concerns of public safety and reduce gun-related violence. Evaluate the reasoning behind this argument and discuss the potential challenges associated with its implementation.
  • The introduction of a comprehensive sex education curriculum in schools is vital to promote safe and responsible behaviour among adolescents. Assess the validity of this argument and discuss the potential benefits and drawbacks of such an initiative.
  • Investing in the development of affordable housing units in urban areas is essential to address the housing crisis and ensure social equity. Analyse the rationale behind this proposal and evaluate its potential impact on both residents and the local economy.
  • The implementation of stricter regulations on the advertising and sale of unhealthy food products will contribute to improving public health and combating the obesity epidemic. Discuss the validity of this argument and its potential implications for businesses and consumers.
  • Increasing funding for arts and culture programs in schools can have a positive impact on students' creativity, critical thinking skills, and overall academic performance. Evaluate the logic behind this proposal and discuss its potential benefits for the education system.
  • Enforcing mandatory financial literacy courses in high schools will equip students with essential skills to make informed decisions and achieve long-term financial stability. Assess the reasoning behind this argument and discuss the potential outcomes of such an educational initiative.
  • The adoption of telecommuting policies in organisations can lead to improved work-life balance, increased employee satisfaction, and enhanced productivity. Analyse the reasoning behind this proposal and discuss its potential advantages and challenges for both employers and employees.
  • Investing in the development of public parks and green spaces in urban areas can have numerous benefits, including improved mental and physical well-being, enhanced community cohesion, and increased property values. Evaluate the soundness of this argument and its potential impact on urban environments.
  • Introducing a universal basic income program can help alleviate poverty, reduce income inequality, and provide individuals with a safety net in an increasingly automated world. Analyse the reasoning behind this proposal and discuss its potential implications for society and the economy.
  • Discuss the impact of social media on interpersonal communication and relationships.
  • To what extent should governments prioritise funding for space exploration?
  • Explain your views on the role of artificial intelligence in shaping the future of work.
  • Discuss the ethical considerations of genetic engineering and its potential impact on society.
  • Explain your perspective on the effectiveness of affirmative action policies in promoting equality.
  • To what extent should the government regulate the sharing economy platforms such as Uber and Airbnb?
  • Discuss the benefits and drawbacks of the globalisation of markets and its impact on local economies.
  • Explain your views on the role of government in promoting sustainable development.
  • Discuss the ethical implications of data privacy and the use of personal information by technology companies.
  • Explain your perspective on the importance of arts and humanities education in today's society.
  • Discuss the impact of climate change on global food security and the role of international cooperation.
  • Explain your views on the ethical considerations surrounding animal testing in scientific research.
  • Discuss the advantages and disadvantages of remote work for employees and organisations.
  • Explain your perspective on the role of the government in regulating the use of social media platforms.
  • Discuss the implications of income inequality and the role of government in addressing this issue.

GMAT practice questions provide an opportunity to develop and refine your critical thinking, analytical, and writing skills. By engaging with a variety of prompts, you can familiarise yourself with different argument structures, evaluate evidence, and effectively express your ideas within the given time constraint.

We have listed ten GMAT sample questions that cover a range of topics and argument types to challenge you to analyse arguments, identify flaws, consider counterarguments, and develop well-structured responses.

GMAT sample question #1

The following appeared in a company memo:

Our new advertising campaign was a great success. The number of customers visiting our website increased by 50% in the first month after the campaign's launch. Therefore, the campaign should be credited with attracting new customers and boosting sales.' Discuss how well-reasoned you find this argument. In your discussion, analyse the argument's assumptions and evidence.

GMAT sample question #2

The following appeared in a research paper:

Implementing a four-day workweek will significantly enhance employee productivity. A survey conducted among companies that adopted a shorter workweek showed a 20% increase in employee output and a 15% decrease in absenteeism rates. Therefore, all organisations should consider adopting a four-day workweek to boost efficiency.' Discuss how well-reasoned you find this argument. In your discussion, analyse the argument's assumptions and evidence.

GMAT sample question #3

The following appeared in a political campaign speech:

Increasing the minimum wage will improve the economy and reduce income inequality. Studies have shown that when the minimum wage is raised, low-income workers experience a rise in their earnings, leading to increased consumer spending. As a result, businesses thrive, unemployment rates decrease, and the wealth gap narrows. Therefore, it is imperative to raise the minimum wage to stimulate economic growth.' Discuss how well-reasoned you find this argument. In your discussion, analyse the argument's assumptions and evidence.

GMAT sample question #4

The following appeared in a blog post:

Investing in early childhood education is crucial for fostering future academic success. Research has consistently shown that children who receive quality early education perform better academically and are more likely to graduate from high school and pursue higher education. Therefore, policymakers should prioritise funding and expanding early childhood education programs.' Discuss how well-reasoned you find this argument. In your discussion, analyse the argument's assumptions and evidence.

GMAT sample question #5

The following appeared in a marketing presentation:

Social media marketing is the most effective strategy for reaching and engaging with the target audience. A survey conducted among consumers revealed that 75% of respondents made a purchase based on social media ads or recommendations from influencers. Therefore, companies should allocate a significant portion of their marketing budget to social media campaigns to maximise their reach and sales.' Discuss how well-reasoned you find this argument. In your discussion, analyse the argument's assumptions and evidence.

GMAT sample question #6

Prompt: Evaluate the argument that increasing taxes on sugary beverages will effectively reduce the prevalence of obesity and improve public health. Discuss the key assumptions and evidence used to support this claim, and identify any potential counterarguments or limitations that may challenge the effectiveness of this approach.

In your analysis, you should consider the relationship between sugary beverage consumption and obesity rates, as well as the potential impact of higher taxes on consumer behaviour. Additionally, assess the strength of the evidence presented to support the argument and examine any potential flaws or biases in the data. Consider alternative approaches to tackling obesity, such as education campaigns or regulations on advertising, and discuss their potential advantages or disadvantages. Ultimately, evaluate the argument's overall persuasiveness and suggest any additional evidence or considerations that could strengthen or weaken the case for increased taxes on sugary beverages.  

GMAT sample question #7

Prompt: Analyse the argument that social media platforms should be held legally responsible for moderating user-generated content. Evaluate the reasons and evidence presented to support this viewpoint, and discuss the potential consequences of imposing legal obligations on these platforms. Consider issues such as freedom of speech, platform liability, and the challenges of content moderation at scale. Additionally, explore alternative approaches to addressing harmful or illegal content online and discuss their potential advantages or drawbacks. Finally, assess the feasibility and potential effectiveness of different regulatory or self-regulatory measures in ensuring a safe and responsible online environment.  

GMAT sample question #8

Prompt: Evaluate the argument that increasing access to higher education will lead to greater socioeconomic mobility and reduce income inequality. Analyse the evidence and reasoning used to support this claim, and consider potential counterarguments or limitations to this approach. Assess the impact of factors such as rising tuition costs, disparities in educational quality, and the relevance of certain degrees in the job market. Explore alternative strategies for promoting social mobility, such as vocational training or income redistribution, and discuss their potential advantages or disadvantages. Ultimately, critically examine the argument's validity and provide recommendations or additional considerations that may strengthen or weaken the case for increasing access to higher education.

GMAT sample question #9

Prompt: Analyse the argument that the use of genetically modified organisms (GMOs) in agriculture is essential for ensuring global food security. Evaluate the evidence and reasoning presented to support this claim, and discuss any potential counterarguments or concerns related to the environmental, health, or socioeconomic impacts of GMOs. Assess the effectiveness of current regulations and labelling practices in addressing these concerns, and explore alternative approaches to achieving food security, such as organic farming or agroecology. Finally, critically evaluate the overall persuasiveness of the argument and provide recommendations or additional considerations that may enhance or challenge the case for GMO usage.

GMAT sample question #10

Prompt: Evaluate the argument that implementing a universal basic income (UBI) would be an effective solution to address poverty and income inequality. Analyse the evidence and reasoning used to support this viewpoint, and consider potential counterarguments or limitations of a UBI system. Assess the economic implications, including funding mechanisms and potential effects on workforce participation and productivity. Explore alternative policies or approaches to poverty alleviation, such as targeted welfare programs or job creation initiatives, and discuss their potential advantages or disadvantages. Ultimately, critically examine the argument's validity and provide recommendations or additional considerations that may strengthen or weaken the case for implementing a universal basic income.

GMAT essay topics

Some common topics that may be covered in the GMAT essays include:

  • The research indicates that the success of a shopping mall is directly related to its location. Therefore, we should build a new shopping mall in the small town of Oakville, which has a growing population and no competition from other shopping malls. Discuss how well-reasoned you find this argument.
  • To remain competitive, companies need to embrace new technologies, including artificial intelligence and automation. Employees who resist these changes are hindering progress and putting their jobs at risk.' Evaluate the argument presented in the editorial.
  • Our company should offer tuition reimbursement to all employees who want to pursue a degree or certification. This will increase employee loyalty, productivity, and skill level, which will benefit the company in the long run.' Discuss the validity of this recommendation.
  • Our new cleaning products are environmentally friendly and safe for use around children and pets. You can trust our products to not only clean effectively, but also protect your family and the planet. Evaluate the claims made in the advertisement."
  • The city should ban all smoking in public parks. Secondhand smoke is a health hazard to all park visitors, and it is unfair to subject non-smokers to this danger. Evaluate the argument presented in the letter.

Also read: TOEFL sample essays

  • It is better to be practical than to be creative when it comes to problem-solving.
  • Governments should prioritise funding for space exploration over other scientific endeavours.
  • The widespread use of social media has had a negative impact on society.
  • The pursuit of happiness should be considered a fundamental human right.
  • The use of technology has had a more positive than negative impact on society.
  • Businesses have a responsibility to prioritise environmental sustainability over profit.
  • The government should provide free healthcare for all citizens.
  • The increasing popularity of e-books will eventually lead to the end of traditional bookstores.
  • Individuals have a responsibility to prioritise their own health and well-being over their career ambitions.
  • The benefits of globalisation outweigh the drawbacks.

Also Read: GMAT Self-Study Plan

Tips for a high-scoring performance

Here are some expert tips to help you achieve a high-scoring performance on the GMAT essay.

The first step to writing a successful GMAT essay is to understand the prompt and the task. Take the time to read the prompt carefully and make sure you understand what is being asked of you. Pay close attention to the specific instructions, such as the length requirement, the number of examples you need to provide, and the time limit.

Before you start writing, take a few minutes to plan your essay. This will help you organise your thoughts and ensure that your essay is well-structured and easy to follow. Create an outline that includes your main points and supporting evidence, and make sure that your essay has a clear introduction, body, and conclusion.

For the Analysis of an Argument task, carefully analyse the argument presented in the prompt. Identify the assumptions and evidence used to support the argument, and consider whether these are valid and sufficient. For the Analysis of an Issue task, consider both sides of the issue and identify the strengths and weaknesses of each argument.

To support your arguments, use specific examples from your own experiences or the world around you. These examples should be relevant and support the points you are making in your essay. Be sure to explain how each example supports your argument.

The GMAT essay is not a test of your writing ability per se, but rather a test of your ability to analyze and evaluate arguments and express your ideas in a clear and concise manner. Use simple and straightforward language, avoid overly complex sentences or jargon, and be sure to proofread your essay for grammatical errors and spelling mistakes.

The GMAT essay should follow a standard essay format, with a clear introduction, body, and conclusion. Your introduction should provide an overview of your argument, while your body should provide supporting evidence and examples. Your conclusion should summarise your main points and restate your thesis statement concisely.

Like any skill, the ability to write a successful GMAT essay takes practice. Take advantage of the practice exams and sample essays provided by GMAC, the organisation that administers the GMAT exam. Consider working with a tutor or taking a prep course to help you improve your essay writing skills.

Articles related to GMAT Exam

  • GMAT Preparation Books
  • GMAT Grammer
  • GMAT Integrated Reasoning Tips
  • Preparing for MBA Without GMAT  
  • GMAT Vocabulary
  • GMAT Sentence Correction
  • GMAT Critical Reasoning
  • Online GMAT Preparation Tips
  • GMAT Reading Comprehension
  • GMAT Math Formulas
  • GMAT Customized Study Plan
  • GMAT Geometry
  • GMAT Test Resources

The GMAT essay is an important part of the GMAT exam and requires careful preparation and planning. By following these expert tips, you can improve your chances of achieving a high score on the GMAT essay and increase your chances of being accepted into your dream MBA program.

Frequently asked questions

The GMAT essay, also known as the Analytical Writing Assessment (AWA), is a section of the GMAT exam that assesses a candidate's ability to think critically and communicate ideas in written form.

The GMAT essay is scored on a scale of 0-6 in half-point increments. Two independent readers will score the essay, and if their scores differ by more than one point, a third reader will be brought in to resolve the discrepancy.

To prepare for the GMAT essay, candidates can practise writing essays using free GMAT essay prompts available online, develop a plan before writing, use specific examples to support their arguments and practise good grammar and syntax.

author avatar

Study Abroad Expert

Stay tuned with us for more updates on studying abroad.

Disclaimer: The views and opinions shared in this site solely belong to the individual authors and do not necessarily represent t ...Read More

The ultimate guide to mastering the GMAT exam

The ultimate guide to mastering the GMAT exam

GMAT vs GRE - Which is easier?

GMAT vs GRE - Which is easier?

6 best GMAT prep books that students swear by

6 best GMAT prep books that students swear by

GMAT vs CAT: Key differences and similarities

GMAT vs CAT: Key differences and similarities

Mastering the GMAT integrated reasoning section: Expert tips

Mastering the GMAT integrated reasoning section: Expert tips

How to prepare for an MBA program without the GMAT

How to prepare for an MBA program without the GMAT

gmat essay question examples

  • Exam Prep >
  • Prepare for Business School >
  • Business School & Careers >
  • Explore Programs >
  • Connect with Schools >
  • How to Apply >
  • Help Center >

Every journey needs a plan. Use our Career Guide to get where you want to be. 

Trending section, help schools discover you through the gmac™ gradselect search – for free, your 2023 guide: how to write an mba resume that stands out, meet your business school match at a gmac tours event.

  • About the Exam
  • Register for the Exam
  • Plan for Exam Day
  • Prep for the Exam
  • About the Executive Assessment
  • Register for the Executive Assessment
  • Plan for Assessment Day
  • Prepare for the Assessment
  • NMAT by GMAC

Related Content

My gmat score paid for my mba: how to unlock your scholarship, why top business schools admit mostly gmat™ applicants, the gmat™ exam gets you business ready.

  • Shop GMAT Focus Official Prep
  • About GMAT Focus Official Prep
  • Prep Strategies
  • Personalized Prep Plan
  • GMAT Focus Mini Quiz
  • Executive Assessment Exam Prep
  • NMAT by GMAC Exam Prep

GMAT™ Official Practice: 5 Benefits of the New and Improved GMAT Prep Experience

How to improve your gmat™ exam score, prepare for business school.

  • Business Fundamentals
  • Skills Insight

Your Guide to Applying for an MBA as a Working Professional [mba.com Insider]

Should i get an mba how to know you’re ready, undergrad’s guide to finding your best fit business master’s program [mba.com insider], business school & careers.

  • Why Business School
  • Student Experience
  • Business Internships
  • B-School Go
  • Quiz: Are You Leadership Material?
  • MBA Return on Investment (ROI) Calculator
  • Estimate Your Salary
  • Success Stories
  • Diversity and Inclusion
  • Women in Business

What is the ROI of an MBA?

Why you're not getting promoted (and what to do about it), business school as a vehicle for doing good, explore programs.

  • Top Business School Programs
  • Quiz: Which Post Graduate Program is Right for You?
  • Quiz: Find the Best Program for Your Personality
  • Business School Rankings
  • Business Master's Programs
  • MBA Programs
  • Study Destinations
  • Find Programs Near Me
  • Find MBA Programs
  • Find Master's Programs
  • Find Executive Programs
  • Find Online Programs

GMAT Accepting Programs Around the World

Mim vs mba: what’s the difference, connect with schools.

  • About GradSelect
  • Create a GradSelect Profile
  • Prep Yourself for B-School
  • Quiz: Can You Network Like An MBA?
  • Events Calendar
  • School Events
  • GMAC Tours Events
  • In-Person Events
  • Online Events

The MBA Tour: Inside the GMAT Podcast

How to apply.

  • Apply to Programs
  • The Value of Assessments
  • Admissions Essays
  • Letters of Recommendation
  • Admissions Interviews
  • Scholarships and Financing
  • Quiz: What's Your Ideal Learning Style?

MBA Essays: Everything You Need to Know

Tell your story: personal branding for mba applications, five to do's for your mba interview, help center, what to do if you regret your college major, why it’s better to be a career generalist in the long run, how can b-school help me with my nonprofit career.

  • Register for the GMAT
  • Create Account

gmat essay question examples

Total: $0.00

Sample questions, quick links.

Before you register  for the GMAT Focus Edition, get a preview of the types of questions you’ll encounter on the exam. 

Remember: You can learn more about section and question type at Exam Content .

Sample Questions by Section

Quantitative reasoning.

Problem Solving Question Directions: Solve the problem and indicate the best of the answer choices given. Question: If a certain wheel turns at a constant rate of x revolutions per minute, how many revolutions will the wheel make in k seconds? (A) 60 kx (B)  kx (C) x ÷ k (D) x ÷(60 k ) (E) kx ÷60 Answer:  (E)

Verbal Reasoning

Reading Comprehension Question Directions: The questions in this group are based on the content of a passage. After reading the passage, choose the best answer to each question. Answer all questions following the passage on the basis of what is stated or implied in the passage. Question: Schools expect textbooks to be a valuable source of information for students. My research suggests, however, that textbooks that address the place of Native Americans within the history of the United States distort history to suit a particular cultural value system. In some textbooks, for example, settlers are pictured as more humane, complex, skillful, and wise than Native Americans. In essence, textbooks stereotype and depreciate the numerous Native American cultures while reinforcing the attitude that the European conquest of the New World denotes the superiority of European cultures. Although textbooks evaluate Native American architecture, political systems, and homemaking, I contend that they do it from an ethnocentric, European perspective without recognizing that other perspectives are possible. One argument against my contention asserts that, by nature, textbooks are culturally biased and that I am simply underestimating children's ability to see through these biases. Some researchers even claim that by the time students are in high school, they know they cannot take textbooks literally. Yet substantial evidence exists to the contrary. Two researchers, for example, have conducted studies that suggest that children's attitudes about particular cultures are strongly influenced by the textbooks used in schools. Given this, an ongoing, careful review of how school textbooks depict Native Americans is certainly warranted. Which of the following would most logically be the topic of the paragraph immediately following the passage? (A) Specific ways to evaluate the biases of United States history textbooks (B) The centrality of the teacher's role in United States history courses (C) Nontraditional methods of teaching United States history (D) The contributions of European immigrants to the development of the United States (E) Ways in which parents influence children's political attitudes Answer:  (A)

Critical Reasoning Question Directions: For this question, select the best of the answer choices given. Question: Mall Owner: Our mall’s occupancy rate is so low that we are barely making a profit. We cannot raise rents because of an unacceptably high risk of losing established tenants. On the other hand, a mall that is fully occupied costs about as much to run as one in which a rental space here and a rental space there stands empty. Clearly, therefore, to increase profits we must sign up new tenants. Which of the following, if true, most seriously weakens the argument? (A) The mall’s operating costs could be cut by consolidating currently rented spaces in such a way that an entire wing of the mall could be closed up. (B) The mall is located in a geographic area in which costs incurred for air-conditioning in the hot summers exceed those incurred for heating in the mild winters by a wider margin. (C) The mall’s occupancy rate, though relatively low, has been relatively stable for several years. (D) The mall lost tenants as a result of each of the two major rent increases that have occurred there. (E) None of the mall’s established tenants is likely to need additional floor space there in the foreseeable future. Answer:  (A)

Data Insights

Data Sufficiency Question Directions: This data sufficiency problem consists of a question and two statements, labeled (1) and (2), in which certain data are given. You have to decide whether the data given in the statements are sufficient for answering the question. Using the data given in the statements, plus your knowledge of mathematics and everyday facts (such as the number of days in July or the meaning of the word counterclockwise), you must indicate whether:

  • Statement (1) ALONE is sufficient, but statement (2) alone is not sufficient to answer the question asked.
  • Statement (2) ALONE is sufficient, but statement (1) alone is not sufficient to answer the question asked.
  • BOTH statements (1) and (2) TOGETHER are sufficient to answer the question asked, but NEITHER statement ALONE is sufficient to answer the question asked.
  • EACH statement ALONE is sufficient to answer the question asked.
  • Statements (1) and (2) TOGETHER are NOT sufficient to answer the question asked, and additional data specific to the problem are needed.

Question: What is the monthly rent for a certain apartment? (1) The monthly rent per person for 4 people to share the rent for the apartment is $375. (2) The monthly rent per person for 4 people to share the rent of the apartment is $125 less than the monthly rent per person for 3 people to share the rent. (A) Statement (1) ALONE is sufficient, but statement (2) alone is not sufficient. (B) Statement (2) ALONE is sufficient, but statement (1) alone is not sufficient. (C) BOTH statements TOGETHER are sufficient, but NEITHER statement ALONE is sufficient. (D) EACH statement ALONE is sufficient. (E) Statements (1) and (2) TOGETHER are NOT sufficient. Answer: (D) For the other four question types, click for samples of each question type to open up in an interactive pop-up window. Please note: these sample questions are built to simulate the actual test interface, and therefore, are not optimized for mobile devices.

  • Multi-Source Reasoning
  • Table Analysis
  • Graphics Interpretation
  • Two-Part Analysis
  • Skip to content

GMAT Prep Courses & Tutoring

Free GMAT Practice Questions

Practice makes perfect—or at least that's how the old saying goes—and it certainly applies to our free GMAT practice questions. Our Free Practice Questions are designed to give you the thorough understanding of how to go about solving a problem that you crave. Our thorough explanations show you what to expect from each GMAT question, detailing question-specific hurdles and common traps. Thankfully, our practice questions provide a wide variety of question types spanning across all sections, securing an abundance of insight-turned-strategy you can implement on test day to turn into high-scoring gold.

Manhattan Review prides itself in providing quality free practice questions to all prospective students, so please take a crack at the 52 free questions we have to offer as a courtesy to all GMAT learners. In the end, the only way to find out where you need your score to be is to discover where it currently is. Take advantage of this free resource that's sure to help you along your way to a high score.

You have not answered any question so far. You can answer all questions in a row (click on "All Questions") or only all questions of a particular section (click on that Section) or a single selected question (click on that Question).

GMAT is a registered trademark of the Graduate Management Admission Council (GMAC), which is unaffiliated with and does not endorse this website.

Enjoy this post? Rate it!

GMAT essay topics

GMAT essay topics | 3 examples included to ace the exam!

The gmat essay topics are distinct from most essays you’ve written in college. it involves critical thinking skills, how well you articulate the matter without picking a side, and so on. read on to know more, table of contents, all you need to know about the gmat essay section, gmat essay topics | things to keep in mind, example 1: it is an excerpt from the grocery store’s business plan, example 2: the publisher of the mercury, a weekly newspaper, issued the following announcement, example 3: protesters at waymarsh state college, #1. time your gmat essay practice, #2. write your gmat essays, #3. assess the quality of your gmat essays, key takeaways.

The GMAT essay section provides a one-paragraph prompt containing a specific argument. This document includes practice questions to help you improve your essay section , help you analyze how well you have reasoned the statement, and so on. While attempting the essay section, you have to keep certain aspects in mind. To help you nail the exam, here are some GMAT essay topics that you must practice positively. 

Before proceeding ahead with the GMAT essay topics, here are a few things you need to keep in mind –

  • Ensure to analyze the argument’s logic and the incorporation of proof in the assertion during your discussion.
  • Consider the debatable assumptions that underpin thoughts and the possible theories or counterpoints that might undermine the conclusion. 
  • You can also talk about what kind of evidence would help or hurt the argument. 
  • Additionally, consider what changes to the debate would make it more logical and what would help you help assess its outcome.

GMAT essay topics |Examples that you must know of

Now that we’ve covered what the essay task requires, let’s first look at some sample GMAT essay topics from the official list.

No clear categories of provokes affect your assessment , but for a representative sample. It’s worth noting that the construction of each assertion does not fall within these boundaries; political prompts can employ the same flawed confrontational strategies as business or health and science prompts, for example.

Practicing GMAT essay topics before the actual exam

‘Even though we introduced a pharmacy segment to our supermarket this year, our overall sales have risen by 20%. The main concern of the customer is the convenience provided by one-stop shopping. Adding a clothing department and a vehicle supplies and repair shop is the most certain way to boost our profits in the coming years. In the coming years, we must also plan to add different departments and assistance, including a restaurant and a landscape shop. We will have a competitive edge over other local retailers since we are the only store in the district that offers such a wide range of services.’

Start writing answers wherein you discuss one or more clashing framework explanations and explain how the summaries can compensate for the proof stated in the proposition. Finally, look for logical flaws in the ending, facts, and rationale.

GMAT essay topics | 3 examples included to ace the exam!

‘Since the launch of a competing lower-priced newspaper, The Bugle, five years ago, The Mercury’s circulation has dropped by 10,000 readers.” The best way to encourage more people to read The Mercury is to lower its price less than that of the Bugle until circulation returns to pre-recession levels. Mercury’s increased circulation will entice more businesses to purchase ad space in the paper.’

Attempt writing responses in which you deliberate one or more contending framework explanations and describe how your descriptions can account for the evidence stated in the argument. Finally, examine the conclusion, evidence, and reasoning for logical flaws.

The following appeared in a local newspaper’s editorial section:

‘This past winter, 200 Waymarsh State College students marched to the state’s capital to hold rallies against proposed budget cuts to various state college programs.  On the other hand, the other 12,000 Waymarsh students either remained on campus or left for the winter holidays, indicating that they were less concerned with their education. Because the non-protesters outnumber the protesters, they represent the state’s college students more. As a result, the state legislature should disregard the protesting students’ pleas.’

Start to write solutions in which you clarify a few conflicting arrangement reasons and illustrate how the overviews can make up the difference for the verification outlined in the hypothesis. Finally, take a glance for technical inaccuracies in the conclusion, factual information, and reasoning skills.

GMAT essay topics | Tips to ace the section!

The tips that follow will assist you in preparing for any GMAT essay topics you may encounter on test day –

When practicing GMAT essay topics, remember that you have a strict 30-minute time limit for your essay.

You should start writing all of your practice essays on a compute r to simulate the situations of the computer-based GMAT.

After you’ve completed your practice GMAT essays, try to grade them using the 6-point grading set of guidelines and by correlating your writing to the GMAC’s sample essay.

  • The GMAT essay is unlike most of the essays you’ve written in college. 
  • The GMAC provides a list of the most formal GMAT essay topics asked in the exam. 
  • Ensure to practice the essay topics every day. 
  • The more you practice, the better you get at it. 
  • Answer mock GMAT essays. It will help you grasp the technique of writing excellent essays. 
  • Ensure that you articulate the matter well. Use your critical thinking skills for the same. 

If you need assistance in your GMAT preparation, we are here for you. Get in touch with our experts today!

Liked this blog? Then read the blog on the GMAT score chart | 5 points to enhance your exam strategy!

Question 1: What happens if a student fails the GMAT?

Answer: A student can take the GMAT up to five times yearly. As a result, even if you fail once, you will be given another chance.

Question 2: What is the most effective method for preparing for the GMAT?

Answer: The most effective way to prepare for the GMAT is to continue attempting and solving practice papers. We also recommend that you thoroughly examine the most common GMAT questions.

Question 3: Can I find the GMAT essay topics for practice online?

Answer: You can find the GMAT essay topics for practice online. As a matter of fact, you can visit the GMAC site for mock papers. 

How useful was this post?

Click on a star to rate it!

Average rating 0 / 5. Vote count: 0

No votes so far! Be the first to rate this post.

gmat essay question examples

People also liked

IELTS success

IELTS success| Tips to master each section

IELTS Grammar

Importance of IELTS grammar | Tips and tricks

TOEFL and IELTS

TOEFL and IELTS | Role of contextual & academic words

simulated tests

Simulated tests | Opportunity to overcome challenges

GRE exam structure

TOEFL, IELTS, & GRE exam structure | Key differences 2024!

GMAT exam

Ace the GMAT exam | Navigate quant & verbal difficulties!

Leave a reply cancel reply.

Your email address will not be published. Required fields are marked *

Please enter an answer in digits: 4 × two =

Start your journey with iSchoolPrep

Need help with your Test Preparations? Contact Us for more details

gmat essay question examples

Inquire Now

Get e-books, expert guidance, live classes and more....

GMAT Prep Online Guides and Tips

2000+ gmat sample questions: practice every question type.

gmat essay question examples

Practice questions are an essential part of any GMAT prep. But how do you find the best GMAT sample questions?

In this article, I’ll address what to look for in GMAT practice questions, the pros and cons of both official and unofficial GMAT test questions, and tips on creating an effective GMAT study plan using these resources.

What to Look for in GMAT Sample Questions

There are a wide variety of GMAT sample questions out there. How do you know if you’re using quality practice ones? These are a few of the characteristics of good GMAT practice questions, whether they’re official or unofficial.

#1: Same Format and Question Types as the Real GMAT

Once you start taking GMAT practice tests, you’ll get a feel for how GMAT questions should look: how long they are, what they look like on the screen, the topics they tend to cover, and the style in which they’re written, for example. In every section, there will also be a mix of question types: the quant section always includes both data sufficiency and problem-solving questions, while the verbal section always includes a mix of sentence correction, reading comprehension, and critical reasoning questions.

The GMAT example questions you choose as you prepare for the test should be as similar as possible to real questions written by the Graduate Management Admissions Council (GMAC). The closer you can get to simulating real testing conditions, including everything from visuals to question content, the more comfortable you’ll feel on the day of the test. Also, answering GMAT practice questions that are similar to those you’ll encounter on exam day will help you gauge your progress and potential GMAT score range accurately.

Not sure how or what to study? Confused by how to improve your score in the shortest time possible? We've created the only Online GMAT Prep Program that identifies your strengths and weaknesses, customizes a study plan, coaches you through lessons and quizzes, and adapts your study plan as you improve.

We believe PrepScholar GMAT is the best GMAT prep program available , especially if you find it hard to organize your study schedule and don't want to spend a ton of money on the other companies' one-size-fits-all study plans.

     Improve Your GMAT Score by 60 Points, Guaranteed     

#2: Computerized (and Hopefully Adaptive)

The GMAT is a computerized adaptive test (CAT), which means it adapts to the test taker’s skill level in real time to determine both their score and the mix of questions they’re given. Ideally, GMAT sample questions should follow this format, so you can be best prepared for what you’ll see and experience on test day. Practice resources in CAT format will usually advertise that fact prominently. If they don’t, they’re likely not in the format of the official GMAT.

Obviously not every resource you use will be in CAT format—for example, if you’re using a book to do drills on specific question types. But a sizable portion of your prep should be done using practice questions in CAT format.

#3: Accurate Difficulty Levels

The GMAT always includes a mix of easy, medium and difficult questions. How many you receive of each will depend partly on the computerized adaptive test: You’ll start with ‘medium’ questions, and if you answer those correctly, you’ll be given more difficult questions on average; if you answer them incorrectly, you’ll get easier questions on average. However, every test taker will receive some questions at all three difficulty levels.

Your selection of practice GMAT questions should be at the same difficulty level as the real GMAT test questions. As you prepare, you’ll be able to tell if your sample GMAT questions seem too easy or too difficult on average. Practicing with either too-difficult or too-easy questions might give you an inaccurate picture of your probable ultimate GMAT score, and could also leave you underprepared for the real GMAT.

#4: Organized by Skill and/or Difficulty

You should target your weaknesses when completing GMAT practice questions. You may struggle more with data sufficiency questions than with problem-solving questions on the quant section, for example, or you may do well on most geometry questions but wrestle more with trigonometry and algebra. Alternatively, you may have mastered easy questions across the board and might need to focus on medium and/or difficult questions instead.

Resources that offer customizable sets (based on question type, skills tested, and difficulty level) of GMAT practice questions, or that separate the questions according to those categories, will help you study more effectively and in a more organized fashion than ones that aren’t labeled according to type or that are randomly compiled. The more specifically each question is categorized, the more effectively you will be able to form a study plan that hones in on your specific problem areas.

Computerized adaptive practice questions will help you prepare most effectively for the GMAT.

Official GMAT Practice Questions

The Graduate Management Admissions Council (GMAC) releases a variety of practice resources that use official GMAT sample questions, either written by the test makers themselves or adopted from retired exams. For each resource, I’ll go over what’s included, its strengths and weaknesses, and the best way to make use of it in your prep.

#1: GMATPrep Software

The official GMATPrep Software is the best resource for GMAT practice questions available. It contains two full-length simulated computerized adaptive tests . Everything is written by GMAC, so you know you’re getting the real thing. You can take the two practice tests as many times as you like, but you’ll likely repeat some of the same questions if you take each one more than once.

You can customize your sets of practice questions, choosing 1-15 questions of each type (critical reasoning, sentence correction, and reading comprehension, for verbal) and difficulty (easy, medium, or difficult). In addition to the questions on the two practice tests, there are 90 additional practice questions available on the software.

The software also includes a detailed breakdown of GMAT question types and strategies, as well as a review of skills you’ll need for the quant section.

  • It’s free for registered users of mba.com. Just create an account.
  • The practice questions are identical in length, format, style, content, and visuals to what you’ll see on the GMAT.
  • Answer explanations are step-by-step and in-depth, and you can go back and review or drill questions you previously got wrong as many times as you like.
  • Detailed performance reviews break down what you’re getting wrong so you can hone in on your weak spots, including specific question types and skill sets
  • Not much! The GMATPrep exams are great GMAT example tests. There aren’t many resources that can beat official questions under simulated testing conditions, and it’s free.
  • However, there are only two full-length practice tests, so you’ll likely need to supplement these practice questions with other resources.
  • Also, while the quant review is great, there isn’t a similar resource available on the software for the verbal section.

How to Use It:

  • It’s best to start off your GMAT prep by taking one of the two full-length simulated tests on the GMATPrep software. Since the practice questions and score report are so realistic, the results will serve as a good barometer of your starting point.
  • After you complete the rest of your prep with other practice questions, you should take the second GMATPrep practice test a few weeks before you take the GMAT. This will give you a good idea of where you stand before exam day.

#2: GMATPrep Question Pack 1

An official addition to the GMATPrep Software, the GMATPrep Question Pack contains 404 additional official questions.

You can create custom sets of practice questions of any length based on your needs. Each set of questions can be completed in either ‘exam mode’ (timed, under simulated test conditions) or ‘study mode’ (untimed). The Question Pack will cost you $29.99 to download.

  • This resource contains practice questions that aren’t available in any other official prep materials.
  • Your performance reviews for these practice questions will be integrated into your overall progress reports in the GMATPrep Software , allowing you to get an even more accurate picture of what your needs are in terms of prep.
  • There isn’t much of a downside to official questions as a supplement to your overall prep.
  • Once you’ve taken a few practice tests and figured out your weaknesses, create custom practice question sets that target your difficult spots (particular sections, question types, or question difficulty levels). These will serve as drills to beef up your skills in those areas.

GMAT customizable question banks allow you to work on your timing.

#3: The GMATPrep Exam Collection

These are official additions to the GMATPrep Software. You can only get them once you’ve downloaded the original free software.

Each Exam Pack contains 90 additional practice questions (30 quantitative, 45 verbal, and 15 integrated reasoning), as well as two full-length computerized adaptive GMATs. Each Exam Pack costs $49.99.

Want to improve your GMAT score by 60 points?

We have the industry's leading GMAT prep program. Built by Harvard, MIT, Stanford, and Wharton alumni and GMAT 99th percentile scorers, the program learns your strengths and weaknesses and customizes a curriculum so you get the most effective prep possible.

Try PrepScholar GMAT for 5 Days Risk-Free.

  • In-depth diagnostics will let you know how you’re doing in comparison to your peers on every subsection of the test , as well as how your pacing could improve on each question type.
  • Answer explanations offer step-by-step, detailed analyses of all practice questions.
  • Not much (official GMAC questions are always helpful), except that they’re a bit pricey.
  • These practice questions are a great first step after you take an initial diagnostic practice test, particularly if you’re still unsure of what exactly your weak spots are.
  • You can use the two full-length CAT GMATs to gauge your progress throughout your prep. Alongside the official GMATPrep Software, you’ll have four full-length tests, so you can space them out to regularly check up on how your score is improving.

#4: GMAT Write

GMAT Write is a fairly new tool released by the makers of the GMAT, meant to help you with the analytical writing assessment. It includes two unique sample prompts for the analytical writing assessment section.

GMAT Write will time your essays (30 minutes each, just like on the real exam) and score them in real time according to the GMAC rubric. It costs $29.99 to download.

  • The visuals and experience of writing the essay are highly realistic.
  • The sample questions are written by GMAC, so you know they’re reliable.
  • GMAT Write scores your essays according to the same criteria used on the real GMAT— ‘analyzes the issue,’ ‘supports ideas,’ ‘organizes a coherent idea,’ and ‘language control’ — in real time. It’s the only official GMAC tool that will score your sample essays.
  • There are only two unique exam prompts included, which isn’t necessarily a lot of practice for $29.99.
  • You won’t get any detailed feedback , just scores in each of the four categories used to assess GMAT essays.
  • Use it alongside other practice tests. Many other resources only have integrated reasoning, verbal and quantitative practice questions, so adding a prompt from GMAT Write to your practice test will help you more closely simulated the experience of the actual GMAT.

GMAT Write will help you prep for the Analytical Writing Assessment.

#5: The  GMAT Official Guide 2018

The GMAT Official Guide 2018  is a comprehensive resource for GMAT prep. Included in your purchase ($19.95) is access (both online and in print) to over 900 official practice questions, access to an accompanying site where you can customize sets of practice questions, and online videos with plenty of GMAT tips and strategies.

  • The questions are written by GMAC, so they’re high quality.  The online practice questions mimic the visuals of the real GMAT.
  • The practice questions are organized in order of difficulty.
  • The answer explanations and introduction sections (where key GMAT concepts get broken down) are too complex for non-advanced students. This is especially true for the quant and sentence correction questions.
  • The online question bank only allows you to save ten practice sessions at a time, so if you plan on doing more, you’ll need to delete them before starting a new session.
  • The online question bank is great for regular practice sessions. The question sets are customizable, and you can review them at any time.
  • The book itself is a good resource to start your practice with after using the GMATPrep software as an initial diagnostic tool.

#6: IR Prep Tool

The Integrated Reasoning (IR) Prep Tool was introduced in 2014 to aid students specifically in the integrated reasoning section.

The tool includes 48 integrated reasoning questions total: 10 graphics interpretation questions, 15 multi-source reasoning questions, 15 two-part analysis questions, and eight table analysis questions.

You can create customized practice question sets, focusing on all question types or just one. You can also set the difficulty level for each practice question set. Once downloaded, you have unlimited use of the prep tool for six months. The IR Prep Tool can be downloaded for $19.99.

  • A detailed ‘help’ section breaks down each integrated reasoning question type for you and gives tips on how to approach it.
  • A time management tool tracks your average time on each question type and difficulty level, allowing you to work specifically on your pacing.
  • The IR Prep Tool doesn’t give you a specific score estimation. Your official IR score will be between 1 and 8, but the IR Prep Tool gives you a score between 0 and 100 based on your performance and time management.
  • There aren’t many resources out there specifically aimed at the integrated reasoning section, a frequent problem area for students, so this is a goldmine for students who have trouble with it.
  • Use it to target and break down the integrated reasoning section into manageable chunks, particularly if you get overwhelmed by the fact that each question has multiple components.

#7: GMAT Focus Quantitative Diagnostic Tool

The GMAT Focus Quantitative Diagnostic Tool consists of a 24-question quantitative test (12 data sufficiency questions and 12 problem-solving questions ) that uses real questions from retired exams. It’s computer adaptive and follows the style and format of the actual GMAT quant section.

You can start and stop the test manually, but you have 45 minutes total to complete the practice questions.

Want to Identify YOUR GMAT Strengths and Weaknesses?

Our proprietary GMAT Diagnostic Assessment creates a customized study plan for you that takes you from registration all the way to test day! It is included with every account and proven to significantly maximize your score .

Get your personalized assessment as part of your 5 day risk-free trial now:

Get Your Free GMAT Diagnostic Assessment Here

When you complete the test, the Focus Tool provides detailed answer explanations and an assessment of your strengths and weaknesses in terms of quant concepts.

Four unique tests are available. One test costs $29.99, and a three-test package costs $79.99.

  • The GMAT Paper Tests are the only other GMAT practice resources that use real retired GMAT questions, making the Focus Tool the only computerized adaptive resource to do so. So you get the best of both worlds with this one: real practice questions from previous GMATs and simulated testing conditions.
  • The detailed assessment of your performance, including a possible score range (more accurate than an exact score, since no one practice test will give you a perfect prediction of your ultimate GMAT score) and a breakdown of your performance by question difficulty, will tell you exactly which quant skills you need to work on before the GMAT.
  • It’s a bit pricey for just 24 questions each.
  • It’s not tied into the other GMAT prep resources, so your results won’t be integrated into your GMATPrep performance reports.
  • After you complete one of the GMATPrep exams, this tool will help you hone in on the quant section . You’ll get an overall idea of your strengths and weaknesses in quant from the GMATPrep Software, but this tool will help you get much more specific about which skills to target in your quant prep.

The Quantitative Focus Diagnostic Tool will help you hone in on the intricacies of the GMAT quant section.

#8: GMAT Paper Tests

The GMAT Paper Tests are real retired GMAT tests , written by GMAC. Each set (I, II, and III) contains three official GMAT tests, answer sheets, and a guide for converting your raw score into a scaled score. They’re available in downloadable PDF form. Each set costs $29.99.

  • Most of the questions in these tests aren’t available in any of the other official GMAT practice tests or resources, so they’re especially good if you want some extra practice. This is a big plus: It’s somewhat difficult to find GMAC-authored questions that don’t overlap with other resources.
  • Official questions are always a big positive in terms of GMAT prep!
  • They’re on paper! You’ll have to score them yourself using the answer sheet, and it’s not computerized (obviously) or adaptive, so it doesn’t simulate any of the testing conditions.
  • No answer explanations are included, just the answers themselves.
  • You may want to order these if you’re a test-taker with a disability that may require an accommodation and using paper tests at home is easier for you during prep time.
  • The GMAT Paper Tests might also serve you well if you’ve used all the other official sample GMAT questions and you want some additional prep. However, don’t rely on them to give you much info on your likely final GMAT score, since they aren’t in the official GMAT format. Use them to work through practice questions in your prep sessions instead.

Best Unofficial GMAT Practice Questions

Official GMAT questions are great, but they aren’t the only GMAT prep materials out there. There are also a variety of free and paid resources that offer quality GMAT practice questions. Here, I’ve included practice tests, books, and online question banks. For each unofficial resource, I’ll address how you can access it, its pros and cons, and the best way to incorporate it into your GMAT prep.

#1: Veritas Prep

You’ll need to create an account to access Veritas Prep’s GMAT Question Bank, but once you do, it’s completely free.

The Question Bank offers hundreds of realistic sample GMAT questions: quant, verbal, and integrated reasoning. It includes example questions of every type within those categories. Before you start your practice session, you can choose which kinds of questions you want to see. With one account, you’ll only see each practice question once.

Veritas Prep doesn’t give you an official estimated score, but it does give you an overall percentile , which measures how you did on a particular question set compared to other students who answered the same practice questions.

The Veritas Prep Question Bank is ideal for regular practice throughout your prep, particularly if you have varying amounts of time (you can do a short or lengthy session) and want to drill certain skill sets or question types . The questions from the bank aren’t in CAT format, but you can access one free computerized adaptive test through Veritas and six paid ones. Veritas Prep questions are some of the closest you’ll find to official GMAC-authored GMAT example questions.

Kaplan has a wide variety of GMAT resources, including in-person and online prep courses, but its GMAT Premier 2018  is its flagship GMAT prep tool. Your purchase includes access to over 1,000 GMAT practice questions, six full-length practice tests (one is in the book, and five are computerized adaptive tests available online), video tutorials, an online question bank with 200 GMAT sample questions and customizable quizzes, and a mobile app that allows you to study on the go.

GMAT Premier also includes a special section of forty advanced quant practice questions and in-depth answer explanations, which can help if you’re looking for a challenge in the quant department. The Kaplan practice questions are a bit harder than the ones in Princeton Review, but the verbal questions in particular still leave something to be desired in terms of difficulty level. The sentence correction questions in particular are easier than real GMAT questions. The Kaplan quant questions fare a bit better in terms of GMAT comparability.

Like Princeton Review, Kaplan is a good resource if you’re looking for a fairly solid overview and breakdown of the exam to start your prep (after you take your first diagnostic GMATPrep test). It’s not going to be as helpful if you’re looking for a comprehensive guide to the GMAT or a massive boost in your score.

#3: Manhattan Prep

With an account, you can take one full-length computerized adaptive Manhattan Prep test for free. Six additional ones are available for $49.99, which gives you one year of unlimited access.

You can take the Manhattan Prep practice GMATs timed or untimed, and you can take complete or partial practice exams depending on your time limit during prep. After you take all six, you can reset the questions to take more. Each test includes an in-depth assessment of your performance that analyzes your strengths, weaknesses, timing, and the difficulty levels you’ve mastered.

Manhattan Prep questions are mostly quite realistic in content and difficulty level. Though the visuals aren’t perfect, they are not too far from what you’ll see on the real exam.

I’d also like to point out one of Manhattan Prep’s best resources for students looking to challenge themselves in the quant section: Their GMAT Advanced Quant book includes strategies for tackling the trickiest quant questions and over 150 realistic practice questions. This is a great resource if you’re already doing well in the quant section and looking to bump yourself up to the next level.

#4: PowerScore

PowerScore’s main claim to fame is its trilogy of GMAT Verbal Bibles: Critical Reasoning, Sentence Correction, and Reading Comprehension, along with the PowerScore Verbal Bible , which covers the entire section. The books are available for $24.99 each or $69.99 as a trilogy.

PowerScore does an excellent job of breaking down each question type within the verbal section (a great resource for non-native English speakers and anyone who struggles with verbal questions), and the sample GMAT questions included in each book are consistent with the length, difficulty, and content of those on the GMAT. Though of course the practice questions here aren’t computerized or adaptive, each book does include access to a companion website that includes extra drills and practice questions.

The main downside to PowerScore is that, while the books offer great strategies and explanations of various question types, there aren’t a substantial number of actual practice questions . The Verbal Bible , for example, contains 31 critical reasoning questions, 77 sentence correction questions, and only eight reading comprehension questions. So while these resources are great reads for test-takers who are looking for in-depth answer explanations, they’re not fantastic for substantial extra practice or regular prep sessions.

The PowerScore Verbal Bibles have excellent grammar guides to help you with sentence correction questions in particular.

#5: 800 Score Test

Providing a username and password will get you one full-length test on 800 Score Test for free. You can download five more for $39.95, which includes a comprehensive quant and verbal review, ten sample essay questions, access to online help, and in-depth video explanations of practice questions.

The visuals at 800 Score Test aren’t much like what you’ll see on the GMAT, but the practice questions themselves are high quality, particularly in the quant section. The verbal practice questions veer a little ‘off’ and aren’t as close to the real GMAT verbal section as the quant questions are, so if you’re especially looking to focus on verbal practice questions, this might not be your best bet.

The 800 Score tests are in CAT format, though, so even though the visuals are a bit lacking, they’re not the worst resource if you only want practice questions that simulate real testing conditions. Their Test Pacer tool lets you know what question you should be on to finish in time, which is very helpful for students who have trouble with consistent pacing.

#6: Princeton Review

Princeton Review’s Cracking the GMAT, available for $21.99 on Kindle, offers a breakdown of all GMAT sections (including integrated reasoning and the analytical writing assessment), along with six full-length computerized adaptive practice exams, over 180 practice questions organized by difficulty level, and drills for each test section. Cracking the GMAT Premier is $7 more and comes with more online resources, like extra practice tests, study strategies, and video tutorials.

Princeton Review offers a decent overview of each of the GMAT sections, though it only offers the grammar and math fundamentals, not anything in the way of more advanced concepts. Indeed, where it misses the mark is mainly in the difficulty of the practice questions: Nearly none of them are as difficult as actual GMAT questions. This is especially true in the critical reasoning and reading comprehension sections. So while Cracking the GMAT may help beginners who aren’t familiar with GMAT basics (especially to start off, since it’s not an intimidating text), it’s not enough for test-takers looking to break 700 or even 650.

#7: GMAT Club

Beyond its helpful GMAT forums, GMAT Club offers a variety of useful GMAT practice materials, including practice tests and question banks.

A few practice tests and question banks are free (no account required), but access to all of their customizable quizzes, bonus questions (over 1,572) and full-length tests will run you at least $79.99 for three months of use.

GMAT Club tests aren’t exactly in CAT format, as they don’t adapt to your performance as you go along, but they do follow CAT ‘logic’ and include a mixture of difficulty levels and question types that is consistent with what you’ll usually encounter in a computerized adaptive test. The quality of the visuals and practice question content is high, with practice questions closely resembling GMAC ones.

Difficult quant questions are one of GMAT Club’s specialties, which is important as many of the other top practice resources (Princeton Review and Kaplan, for example) trend a little easier: They claim that hundreds of their quant questions are at the 700+ scoring level. So if you’re looking to wow your prospective schools with your quant score, this might be a great resource.

#8: Manhattan Review

You have several options for free practice questions and other GMAT prep materials with Manhattan Review. With an account, you can access a fairly high-quality quantitative question bank in PDF ebook form, along with a vocabulary list (good for non-native English speakers) and another free ebook, GMAT in a Nutshell , that serves as an overall guide to the test and top strategies.

The Sentence Correction Guide, downloadable for free, boasts a solid grammar review of the concepts that are more often tested in sentence correction questions, but the sentence correction practice questions themselves aren’t all quite as difficult or complex as the real ones on the GMAT.

Manhattan Review also offers a full-length free practice test, which you can access with an account. It’s a computerized adaptive test. The questions here are good for extra practice, but the difficulty levels and visuals aren’t always consistent with the actual GMAT.

Vocabulary practice is an integral part of GMAT prep for non-native English speakers.

3 Top Tips for Creating a GMAT Study Plan

Once you’ve selected some practice GMAT questions, it’s time to set up a study plan. It’s important not just to find the best prep materials for you, but to use them to your advantage. Here are a few tips for using your prep materials as effectively as possible:

#1: Use the Two Official GMAC Tests on the GMATPrep Software First and Last

You should start your prep with one of the two full-length GMATPrep Software tests. Since they’re computerized adaptive tests with accurate visuals and questions written by GMAC, they’re the best possible indicator of your ultimate GMAT score. The first practice test will give you a good idea of where you stand and how to begin your prep. Unless you purchase more official GMAC practice tests, take the second free test from the software a few weeks before your exam date. This will give you a good idea of your score at the end of it all, and will let you know what to brush up on last-minute before the big day.

#2: Use Different Materials to Build Different Skills

Tailor the practice GMAT materials you select to your particular needs. For example, if you’re strong in quant but weaker in verbal (or a non-native English speaker), you might use Manhattan Prep, the GMAT Focus Quantitative Diagnostic Tool, or GMAT Club practice questions for your quant prep, since they all feature more advanced quant questions. You might start out more slowly in verbal, completing grammar reviews and drills from one of the PowerScore Bibles before delving more into practice questions. Mix and match resources according to your personal GMAT goals.

#3: Note Differences Between the Conditions of Your Practice and the Conditions of the Real GMAT

Not all of your GMAT practice questions need to be (or should be) in computerized adaptive form. Sometimes you’ll need to take your time to work through a particular question, complete a drill, or review concepts. But when you do take a full-length GMAT practice test, stick to the real testing conditions as much as you can: Use double-sided scratch paper, don’t use a calculator on the quant section, take the right amount of time for each section, and take eight-minute breaks between the integrated reasoning and quant sections and before the verbal section.

Some GMAT practice materials allow you to manually time yourself, take longer or unlimited breaks, or stop and start the test. Be mindful of these differences so you don’t learn to rely on conditions (such as a longer break) that won’t exist on the day of your exam.

whats-next-1462747_1280

What’s Next?

Looking for free full-length practice tests in addition to the best practice questions? You can find a list in our guide to the best free GMAT practice tests .

Our complete collection of GMAT practice tests provides a more comprehensive overview, in addition to reviews, of all available GMAT practice tests.

Finally our guide to how long to study for the GMAT will help you set up a time-effective study plan.

Was this helpful? Sign up for FREE GMAT and MBA guides!

Share this:.

  • Click to share on Twitter (Opens in new window)
  • Click to share on Facebook (Opens in new window)
  • Click to share on Google+ (Opens in new window)

gmat essay question examples

Author: Laura Dorwart

Laura Dorwart is a Ph.D. student at UC San Diego. She has taught and tutored hundreds of students in standardized testing, literature, and writing. View all posts by Laura Dorwart

avatar

  • E-mail & Password
  • Notification Settings
  • Global Settings
  • Applicant profile
  • Update status
  • My GMAT info
  • --> My Education -->